Vous êtes sur la page 1sur 69

Intellectual Property Law ►►| 1

INTELLECTUAL PROPERTY LAW


Atty. Alex Ferdinand Fider

INTRODUCTION In exercising freely our self-knowledge (or


intellect), self-consciousness and self-
determination, we become authors of
What is Intellectual Property?
genuinely human activity. Thus, man's
Intellectual property (IP) refers to creations
of the mind, such as inventions; literary and creative works is an extension of his being
artistic works; designs; and symbols, names as a person. They are manifested in his
and images used in commerce: intellectual creations that include literary
“the finer manifestations of human and artistic works, designs, inventions and
achievement". (Prof. William W. Cornish, other intellectual properties. (St. John Paul
Intellectual Property Law: Patents, II, Theology of the Body)
Copyright, Trademarks and Allied Rights
(2007))
Interests to consider:
What is Intellectual Property Law? 1. Intellectual property - an extension of
That branch of law which enables people to the personality of the creator.
earn recognition or financial benefit from It becomes a matter of justice then that an
what they invent or create (examples: inventor or creator of an intellectual
patents, trademarks and copyrights). property has the right to be compensated
or recognized for the labor, intellect and
By striking the right balance between the will contributed in creating that intellectual
interests of innovators and the wider public property. It would be unjust for others to
interest, the IP system aims to foster an benefit from a creator’s time, labor and
environment in which creativity and expenditure.
innovation can flourish. (World Intellectual 2. The legal protection given to inventors
Property Organization (WIPO)) or creators consists of an exclusive right
to undertake an activity (monopoly as an
Intellectual Property Creations: A incentive).
demonstration of Man’s Original 3. On the other hand, it has been said that
Solitude? a monopoly will create economies of
We are distinguished from other living scale because it allows for a more
beings because of our intellectual creativity, efficient production system.
among others.
A monopoly will likewise guarantee security
Man cannot be considered on equal footing of profits which will give incentive to develop
with any other species of living being on or introduce new intellectual properties.
earth. Man alone can cultivate and
transform it to his own needs with his “self- Monopoly Rights
knowledge, self-consciousness and self- This exclusive right encourages
determination.” (St. John Paul II Theology of monopolistic behavior and may not be
the Body (31 October 1979)) ordained to the “common good.”
A monopolist is able to control market prices
No other living being, can rule the earth. and determine the nature of products and
This demonstrates man's original solitude. services it can deliver to the prejudice of the
Intellectual Property Law ►►| 2

consuming public. In this case, there is no Intellectual Property Defined


incentive to bring down costs. The public Intellectual property rights consist of:
will have to pay the monopoly price offered. 1. Copyright and Related Rights;
In this global economy, there is a perception 2. Trademarks and Service Marks;
that foreign corporations use intellectual 3. Geographic Indications;
property law to legitimize monopoly. 4. Industrial Designs;
5. Patents;
IP Code, Section 2 6. Layout-Designs (Topographies) of
Declaration of State Policy. — Integrated Circuits; and
The State recognizes that an effective 7. Protection of Undisclosed Information.
intellectual and industrial property system (IP Code, Sec.4.1)
is vital to the development of domestic and
creative activity, facilitates transfer of Qualificatio Term of Subject
technology, attracts foreign investments, n IP Right matter of
and ensures market access for our monopoly
products. It shall protect and secure the
exclusive rights of scientists, inventors, Patent Novelty 20 years Invention
artists and other gifted citizens to their
intellectual property and creations, Copyright Originality Life of the Literary,
author musical
particularly when beneficial to the people,
+50 years and artistic
for such periods as provided in this Act.
(in works, etc.
The use of intellectual property bears a
general) (expressio
social function. To this end, the State
n of ideas)
shall promote the diffusion of knowledge
and information for the promotion of Trademark No prior 10 years; The mark
national development and progress and s similar marks extendibl with
the common good. and capable e respect to
It is also the policy of the State to of the goods
streamline administrative procedures of distinguishin or services
registering patents, trademarks and g goods or where it is
copyright, to liberalize the registration on services registered/
the transfer of technology, and to used.
enhance the enforcement of intellectual
property rights in the Philippines.
International Obligations (first
In the Philippines, the new Intellectual instruments to emerge)
Property Code recognizes exclusive rights 1. Paris Convention for the Protection
of intellectual property owners. of Industrial Property (revised 1965 &
1980) – patents, utility models, industrial
However, it declares that the use of designs, trademarks, service marks,
intellectual property bears a social function. trade names, and unfair competition
While monopoly right is permitted, there 2. Berne Convention for the Protection
must be recognition of intellectual property's of Literary and Artistic Works (revised
social function. 1948) –“literary and artistic works”

There are limitations in the exercise of these


exclusive rights, including a fixed term and
other qualifying conditions. The royalties
payable should likewise bear this function.
Intellectual Property Law ►►| 3

Two Features Characterize The Treaties: b. Minimum term of copyright is


1. Protection and national treatment for author’s lifetime plus 50 years.
foreigners: (Art. 7)
2. ‘The protection of the rights of authors in
IP Code, Section 3 their literary and artistic works’ (Art. 1)
International Conventions and Reciprocity. — Emphasis of authorship in copyright
Any person who is a national or who is
domiciled or has a real and effective 3. Moral Rights (Art. 6bis)
industrial establishment in a country which is Author is still recognized even if he
a party to any convention, treaty or is no longer the copyright owner.
agreement relating to intellectual property
rights or the repression of unfair competition,
Rome Convention (1961)
to which the Philippines is also a party, or
Extended copyright protection to:
extends reciprocal rights to nationals of the
Philippines by law, shall be entitled to 1. Performers,
benefits to the extent necessary to give effect 2. Producers of Phonograms and
to any provision of such convention, treaty or 3. Broadcasting Organizations.
reciprocal law, in addition to the rights to
which any owner of an intellectual property The Rome Convention secures protection in
right is otherwise entitled by this Act. performances for performers, in phonograms
for producers of phonograms and in
broadcasts for broadcasting organizations.
2. Harmonizing standards of protection
These are also called the related rights.
Other International Agreements
TRIPS Agreement (1994)
1. Agreement on Trade-Related Aspects
of Intellectual Property Rights (TRIPS) The Agreement on Trade-Related Aspects
of Intellectual Property Rights
2. Rome Convention (International
Convention for the Protection of
Performers, Producers of Phonograms Required states to comply with the
and Broadcasting Organizations) provisions of the Berne Convention
3. Patent Cooperation Treaty especially on national treatment and
minimum standards.
4. Madrid Protocol (Trademarks)
Also it called for the protection of computer
LAW ON COPYRIGHT programs and databases. (Art.10)

LAW ON COPYRIGHT
HISTORY Rationale: Author must be able to retain
Modern copyright began in 18th century some control over the dissemination of his
mainly for printed books. At that time, work.
copyright was primarily a statutory right and
for a term provided by law. Otherwise: others would be free to exploit
In the 19th century, copyright extended to his work without paying compensation.
works of arts and drama.
Copyright is the right granted by statute to the
Berne Convention (1886) proprietor of an intellectual production to its
Covered artistic and literary works exclusive use and enjoyment to the extent
1. Minimum standards specified in the statute. (Dissenting opinion of
a. Copyright arise from the moment CJ Davide, Habana v. Robles, 1999)
of creation and no need for
registration (Art. 5(2));
Intellectual Property Law ►►| 4

“Copyright, in the strict sense of the term, is Economic Rights Moral Rights
purely a statutory right. Being a mere essentially linked
statutory grant, the rights are limited to what to the author’s
the statute confers. (Pearl & Dean v. interests in the
Shoemart, 409 SCRA 231, 2003) work as an
expression of an
Merely a statutory right; covers only the individual’s
works falling within the statutory personality.
enumeration.

Copyright protects expressions rather than Moral rights


ideas and information. (IP Code, Secs. 172 include:
and 173) Economic rights
1. Right of
include:
Attributio
Other Rights recognized under the IP Code: 1. Reproduction
n
1. Economic Right (IP Code, Sec.177) 2. Dramatization
2. Right of
2. Moral Right (IP Code, Secs. 193-199) 3. First public
Alteration
distribution
3. Right of
Economic rights are so known because it is 4. Rental
Integrity
essentially through these rights that copyright 5. Public display
(object to
can become a source of income for its owner, 6. Public
any
by selling them or licensing others to perform performance
prejudicial
the acts restricted by the rights 7. Other
distortion)
communicatio
4. Right to
Moral rights, on the other hand, cannot be n to the public
Restrain
transferred to person other than the author of the work.
use of his
of the work, and are essentially linked to the name.
author’s interests in the work as an
expression of an individual’s personality
SUBJECT MATTER
Economic Rights Moral Rights
ORIGINAL WORKS (IP Code, Sec.172)
They enable the These rights Created by the author through his own skill,
creator to obtain make it possible labor and judgment, without directly copying
remuneration from the for the creator to or evasively imitating the work of another.
exploitation of his undertake (Sambar v Levis 2002)
works by third parties measures to 1. Literary
maintain and a. Books, pamphlets, articles and other
protect the writings;
personal b. Periodicals and newspapers;
connection c. Lectures, sermons, addresses,
between himself dissertations prepared for oral
and the work. delivery, whether or not reduced in
writing or other material form;
d. Letters;
Can be transferred or Cannot be 2. Musical
assigned transferred to a. Dramatic or dramatico-musical
person other than compositions; choreographic works
the author of the or entertainment in dumb shows;
work, and are
Intellectual Property Law ►►| 5

b. Musical compositions, with or subsisting copyright upon the original


without words; works employed or any part thereof, or be
3. Artistic construed to imply any right to such use
a. Works of drawing, painting, of the original works, or to secure or
architecture, sculpture, engraving, extend copyright in such original works.
lithography or other works of art;
models or designs for works of art; List of Derivative Works Protected by Law
b. Original ornamental designs or 1. Dramatizations, translations,
models for articles of manufacture, adaptations, abridgments,
whether or not registrable as an arrangements, and other alterations of
industrial design and other works of literary or artistic works;
applied art; 2. Collections of literary, scholarly or
c. Illustrations, maps, plans, sketches, artistic works, and compilations of data
charts and three dimensional works and other materials which are original by
relative to geography, topography, reason of the selection or coordination
architecture or science; or arrangement of their contents. (IP
d. Drawings or plastic works of a Code, Sec.173.1)
scientific or technical character;
4. Photographs, Films Typographical Arrangement
a. Photographic works including works IP Code, Section 174
produced by a process analogous to Published Edition of Work –
photography; lantern slides; In addition to the right to publish granted
b. Audiovisual works and by the author, his heirs, or assigns, the
cinematographic works and works publisher shall have a copyright
produced by a process analogous to consisting merely of the right of
cinematography or any process for reproduction of the typographical
making audiovisual recordings; arrangement of the published edition of
c. Pictorial illustrations and the work.
advertisements
5. Others Economic Rights (IP Code, Sec. 177)
a. Computer programs; and Under the Berne Convention
b. Other literary, scholarly, scientific In addition to the non-transferable moral
and artistic works rights, the Berne Convention recognizes
transferable economic rights enabling
Note: No copyright protection for works of copyright owners to control the following
applied art or industrial design which have activities in relation to their works:
aesthetic or artistic features that cannot be 1. Translation
identified separately from the utilitarian 2. Reproduction
aspects of the article. Functional 3. Public performance and communication
components of useful articles, no matter of dramatic and musical works
how artistically designed, have generally 4. Broadcasting
been denied copyright protection unless 5. Adaptation
they are separable from the useful article.
(Ching v. Salinas, Sr. G.R. No.161295) Modern copyright activity is focused on
economic rights.
DERIVATIVE WORKS
Protected by Copyright To illustrate, the TRIPS Agreement of 1994
IP Code, Section 173.2 require members to provide commercial
Derivative works shall be protected as rental rights in respect of computer
new works: Provided, however, That such programs and films
new work shall not affect the force of any
Intellectual Property Law ►►| 6

Author is the natural person who has


There are 6 major exclusive economic rights created the work.
arising from ownership of the copyright in
any protected work. Section 171.2
The restricted acts for which a license must be Collective work is a work which has been
sought if they are to be lawfully carried out by created by 2 or more natural persons at
a person other than the copyright owner: the initiative and under the direction of
Reproduction Copying another with the understanding that it will
Dramatization Making an be disclosed by the latter under his own
adaptation of the name and that contributing natural
work persons will not be identified. Section
(adaptation right) 171.3
Public Issuing copies of Communication to the public or
Distribution the work to the communicate to the public means the
public making of a work available to the public
(first sale or by wire or wireless means in such a way
distribution right) that members of the public may access
Rental Renting or lending these works from a place and time
the work to the individually chosen by them.
public
(rental/lending Section 171.4
right) Computer is an electronic or similar
Public Display Communicating the device having information-processing
work to the public capabilities, and a "computer program" is
(public a set of instructions expressed in words,
communication codes, schemes or in any other form,
right) which is capable when incorporated in a
Public Performing, medium that the computer can read, or
Performance showing, or playing causing the computer to perform or
the work in public achieve a particular task or result.
(public performance
right) Section 171.5
Other Public lending is the transfer of
communications possession of the original or a copy of a
to the public work or sound recording for a limited
(IP Code, Sec. 177) period, for non-profit purposes, by an
institution the services of which are
available to the public, such as public
library or archive.
The restricted acts may be described as
methods of reproducing the work. Defining
their scope is important for the copyright
owner to determine: Section 171.6
Public performance
1. Kind of licensing arrangement (with the
1. If other than an audiovisual work - is
licensee); and
the recitation, playing, dancing,
2. Action may be taken in respect of
acting or otherwise performing the
infringement of copyright;
work, either directly or by means of
any device or process
IP Code, Section 171
Definition of Terms –
2. If audiovisual work – is the showing
Section 171.1
of its images in sequence and the
Intellectual Property Law ►►| 7

making of the sounds accompanying subdivisions and instrumentalities,


it audible; including government-owned or
controlled corporations as part of his
3. If a sound recording - is the making regularly prescribed official duties.
the recorded sounds audible at a
place or at places where persons
outside the normal circle of a family CRITERIA FOR PROTECTION
and that family’s closest social
Unprotected Subject Matter
acquaintances are or can be present,
irrespective of whether they are or 1. Any idea, procedure, system method or
can be present at the same place and operation, concept, principle, discovery
at the same time, or at different or mere data as such, even if they are
places and/or at different times, and expressed, explained, illustrated or
where the performance can be embodied in a work;
perceived without the need for 2. News of the day and other miscellaneous
communication within the meaning of facts having the character of mere items
Subsection 171.3. of press information; or
3. Any official text of a legislative,
administrative or legal nature, as well as
Section 171.7
Published works means works, which, any official translation thereof. (IP Code,
with the consent of the authors, are made Sec.175)
available to the public by wire or wireless
means in such a way that members of the Government and Copyright
public may access these works from a IP Code, Section 176
place and time individually chosen by Works of the Government –
them: Provided, That availability of such No copyright shall subsist in any work of
copies has been such, as to satisfy the the Government of the Philippines.
reasonable requirements of the public, However, prior approval of the government
having regard to the nature of the work. agency or office wherein the work is
created shall be necessary for exploitation
Section 171.8 of such work for profit. Such agency or
Rental is the transfer of the possession of office may, among other things, impose as
the original or a copy of a work or a a condition the payment of royalties. No
sound recording for a limited period of prior approval or conditions shall be
time, for profit required for the use of any purpose of
-making purposes. statutes, rules and regulations, and
speeches, lectures, sermons,
Section 171.9 addresses, and dissertations,
Reproduction is the making of one (1) or pronounced, read or rendered in courts
more copies of a work or a sound of justice, before administrative agencies,
recording in any manner or form. Section in deliberative assemblies and in
171.10 meetings of public character.
Work of applied art is an artistic creation The Author of speeches,
with utilitarian functions or incorporated lectures, sermons, addresses, and
in a useful article, whether made by hand dissertations mentioned in the preceding
or produced on an industrial scale. paragraphs shall have the exclusive right
of making a collection of his works.
Section 171.11 Notwithstanding the foregoing
Work of the Government of the Philippines provisions, the Government is not
is a work created by an officer or employee precluded from receiving and holding
of the Philippine Government or any of its
Intellectual Property Law ►►| 8

copyrights transferred to it by 3. Audiovisual Work


assignment, bequest or otherwise; nor Copyright - producer, the author of the
shall publication or republication by the scenario, the composer of the music,
government in a public document of any the film director, and the author of the
work in which copy right is subsisting be work so adapted.
taken to cause any abridgment or
annulment of the copyright or to 4. Letters
authorize any use or appropriation of Copyright – writer subject to Art. 723
such work without the consent of the Civil Code
copyright owners.
From the moment of the work’s creation, it
may already contain a notice bearing the
OWNERSHIP OF COPYRIGHT name of the copyright owner, and the year
The IP Code provides that initial copyright of its first publication and, in copies
ownership shall belong to the author of the produced after the creator's death, the year
work. of such death. (IP Code, Sec.192)

In case of works of joint authorship, the co- RELATED RIGHTS


authors shall be the original owners of the
1. Performers
copyright.
Performers are actors, singers,
However, if a work of joint authorship musicians, dancers, and other persons
consists of parts that can be used who act, sing, declaim, play in, interpret,
separately and the author of each part can or otherwise perform literary and artistic
be identified, the author of each part shall work. (IP Code, Sec.202.1)
be the original owner of the copyright in the
part that he has created Performers’ Rights
a. As regards their performances, the
Other Rules on Copyright Ownership right of authorizing:
1. Employee i. The broadcasting and other
In the case of work created by an author communication to the public
during and in the course of his of their performance; and
employment, ownership belongs to: ii. The fixation of their unfixed
a. The employee, if the creation of the performance. (IP Code,
Sec.203.1)
object of copyright is not a part of his
regular duties even if the employee b. The right of authorizing the direct or
uses the time, facilities and materials indirect reproduction of their
of the employer. performances fixed in sound
b. The employer, if the work is the recordings or audiovisual works or
result of the performance of his fixations in any manner or form; (IP
regularly-assigned duties, unless Code, Sec.203.2)
there is an agreement, expressed or c. Subject to the provisions of Section
implied, to the contrary. 206, the right of authorizing the first
public distribution of the original and
2. Commissioned Work
copies of their performance fixed in
GENERAL RULE: Material object - one
the sound recordings or audiovisual
who commissioned
works or fixations through sale or
Copyright – creator
rental or other forms of transfer of
EXCEPTION: if there is a written
ownership; (IP Code, Sec.203.3)
stipulation to the contrary.
Intellectual Property Law ►►| 9

d. The right of authorizing the Right of Producers of Phonograms


commercial rental to the public of the a. The right to authorize the direct or
original and copies of their indirect reproduction of their sound
performances fixed in sound recordings, in any manner or form;
recordings or audiovisual works or the placing of these reproductions in
fixations, even after distribution of the market and the right of rental or
them by, or pursuant to the lending; (IP Code, Sec.208.1)
authorization by the performer; (IP b. The right to authorize the first public
Code, Sec.203.4) distribution of the original and copies
e. The right of authorizing the making of their sound recordings through
available to the public of their sale or rental or other forms of
performances fixed in sound transferring ownership; (IP Code,
recordings or audiovisual works or Sec.208.2)
fixations, by wire or wireless means, c. The right to authorize the commercial
in such a way that members of the rental to the public of the original and
public may access them from a copies of their sound recordings, even
place and time individually chosen after distribution by them by or
by them. (IP Code, Sec.203.5) pursuant to authorization by the
producer. (IP Code, Sec.208.3)
Moral Rights of Performers d. The right to authorize or make
a. right to claim to be identified as available to the public of their sound
the performer of his recordings in such a way that
performances members of the public may access the
except where the omission is sound recording from a place and
dictated by the manner of the a time individually chosen or
use of the performance, selected by them, as well as other
b. right to object to any distortion, transmissions of a sound recording
mutilation or other modification with like effect. (IP Code, Sec.208.4)
of his performances that would
be prejudicial to his reputation
3. Broadcasting Organizations
2. Producers of Phonograms IP Code, Section 202.7
IP Code, Section 202.5 Broadcasting means the transmission by
Producer of a sound recording means the wireless means for the public reception of
person, or the legal entity, who or which sounds or of images or of representations
takes the initiative and has the thereof; such transmission by satellite is
responsibility for the first fixation of the also "broadcasting" where the means for
sounds of a performance or other decrypting are provided to the public by the
sounds, or the representation of sounds. broadcasting organization or with its
consent.

IP Code, Section 202.2 IP Code, Section 202.8


Sound recording means the fixation of Broadcasting organization shall include a
the sounds of a performance or of other natural person or a juridical entity duly
sounds, or representation of sound, other authorized to engage in broadcasting.
than in the form of a fixation incorporated
in a cinematographic or other audiovisual Exclusive Right to Carry Out,
work. Authorize or Prevent:
a. The rebroadcasting of their
broadcasts; (IP Code, Sec.211.1)
Intellectual Property Law ►►| 10

b. The recording in any manner, means the name or designation identifying


including the making of films or or distinguishing an enterprise.
the use of video tape, of their
broadcasts for the purpose of Meanwhile, the scope of a copyright is
communication to the public of confined to literary and artistic works which
television broadcasts of the are original intellectual creations in the
same; (IP Code, Sec.211.2) literary and artistic domain protected from
c. The use of such records for fresh the moment of their creation. Patentable
transmissions or for fresh inventions, on the other hand, refer to any
recording. (IP Code, Sec.211.3) technical solution of a problem in any field
of human activity which is new, involves an
inventive step and is industrially applicable.
DATA PRIVACY ACT
Pearl & Dean v. Shoemart
Data Privacy Act of 2012 (RA 10173), Section Copyright does not extend to structures
4 Scope – themselves.
This Act applies to the processing of all
types of personal information and to any Joaquin v. Drilon
natural and juridical person involved in Ideas, concepts, formats, or schemes in
personal information processing their abstract form clearly do not fall within
including those personal information the class of works or materials susceptible
controllers and processors who, although of copyright registration as provided in PD.
not found or established in the No. 49.
Philippines, use equipment that are
located in the Philippines, or those who Baker v. Selden
maintain an office, branch or agency in But there is a clear distinction between the
the Philippines subject to the immediately book, as such, and the art which it is
succeeding paragraph: Provided, That intended to illustrate.
the requirements of Section 5 are
complied with. The novelty of the art or thing described or
explained has nothing to do with the validity
Habana v. Robles of the copyright. To give to the author of the
The SC said that Respondent’s act of lifting book an exclusive property in the art
from the book of Habana et al substantial described therein, when no examination of
portions of discussions and examples, and its novelty has ever been officially made,
her failure to acknowledge the same in her would be a surprise and a fraud upon the
book is an infringement of Habana et al’s public. That is the province of letters-patent,
copyrights. not of copyright.

Kho v. Court of Appeals


Trademark, copyright and patents are
NATURE OF THE RIGHT
different intellectual property rights that Limitations
cannot be interchanged with one another. 1. Specific limitations (IP Code, Sec.184)
2. Fair Use (IP Code, Sec.185)
A trademark is any visible sign capable of
distinguishing the goods (trademark) or General Defense Against Infringement
services (service mark) of an enterprise and IP Code, Section 185
shall include a stamped or marked container Fair Use of a Copyrighted Work –
of goods. In relation thereto, a trade name Section 185.1
Intellectual Property Law ►►| 11

The fair use of a copyrighted work for of acts which do not reasonable
criticism, comment, news reporting, constitute copyright manner without his
teaching including multiple copies for infringement even if consent,
classroom use, scholarship, research, done without the notwithstanding
and similar purposes is not an consent of the the monopoly
infringement of copyright. copyright holder. granted to the
Decompilation, which is understood owner by the
here to be the reproduction of the code copyright.
and translation of the forms of the
computer program to achieve the inter- It is meant to
operability of an independently created balance the
computer program with other programs monopolies
may also constitute fair use. In enjoyed by the
determining whether the use made of a copyright owner
work in any particular case is fair use, the with the interests
factors to be considered shall include: (P- of the public and of
N-A-E) society.
(a) The Purpose and character of the These limitations, The fair use of
use, including whether such use is of however, should be copyrighted work
a commercial nature or is for non- interpreted in such a for criticism,
profit education purposes; way as to allow the news reporting,
(b) The Nature of the copyrighted work; work to be used in a teaching
(c) The Amount and substantiality of the manner which does (including
portion used in relation to the not conflict with the multiple copies
copyrighted work as a whole; and normal exploitation for classroom
(d) The Effect of the use upon the of the work and does use), research
potential market for or value of the not unreasonably and similar
copyrighted work. prejudice the right purposes is not
The fact that a work is unpublished holder’s legitimate an infringement of
shall not by itself bar a finding of fair use interest. copyright.
if such finding is made upon
consideration of all the above factors.

The above factors are formerly known as MORAL RIGHTS


the three-step test.
Moral Right recognizes certain non-
economic interests which an author may
continue to exercise even though he is no
Limitations on Fair Use (Section longer the owner of the copyright (or the
Copyright (Section 185) material object)
184)
Specific limitations General defense 1. Paternity Right
against To require that the authorship of
infringement the works be attributed to him, in
A privilege, in particular, the right that his name, as far
The law provides for persons other than as practicable, be indicated in a
limitations (“statutory the owner of the prominent way on the copies, and in
fair uses”) on the copyright, to use connection with the public use of his
economic rights of the copyrighted work; (IP Code, Sec.193.1)
authors comprising material in a
Intellectual Property Law ►►| 12

This right shall last during the Such works if not published before
lifetime of the author and in perpetuity shall be protected for 50 years counted
after his death. from the making of the work.
To restrain the use of his name
with respect to any work not of his own 3. Works of applied art - the protection
creation or in a distorted version of his shall be for a period of 25 years from the
work. (IP Code, Sec.193.4) date of making.

2. Right to Integrity (of the work) 4. Photographic works - the protection


To make any alterations of his shall be for 50 years from publication of
work prior to, or to withhold it from the work and, if unpublished, fifty 50
publication (IP Code, Sec. 193.2) years from the making.
To object to any distortion,
mutilation or other modification of, or other 5. Audio-visual works including those
derogatory action in relation to, his work produced by process analogous to
which would be prejudicial to his honor or photography or any process for making
reputation (IP Code, Sec.193.3) audio-visual recordings - the term shall be
50 years from date of publication and, if
Moral right may be waived but the unpublished, from the date of making.
authorship will always be there.

Term Term of Protection for Performances and


Only right of attribution (IP Code, Sound Recordings
Sec.193.1) is in perpetuity. 1. For performances not incorporated in
recordings, 50 years from the end of the
Other moral rights are coterminous with the year in which the performance took
economic rights. (IP Code, Sec.198, as place; and
amended) 2. For sound or image and sound
recordings and for performances
incorporated therein, 50 years from the
DURATION OF COPYRIGHT end of the year in which the recording
took place.
GENERAL RULE: For Original and
Derivative works – lifetime of the author plus
Term of Protection for Broadcast
50 years
The term shall be 20 years from the date
SPECIAL RULES: the broadcast took place. The extended
1. Works of joint authorship - the term shall be applied only to old works with
economic rights shall be protected subsisting protection under the prior law.
during the life of the last surviving author
and for 50 years after his death.
2. Anonymous or pseudonymous works ABS-CBN v. Philippine Multi-Media
The copyright shall be protected for 50 System
years from the date on which the work The must-carry rule falls under the
was first lawfully published “limitations of copyright” provided under
Where, before the expiration of the Sec. 184 (h) of the IP Code, which states
said period, the author's identity is that “the use made of a work by or under
revealed or is no longer in doubt, the the direction or control of the Government,
general rule shall apply xxx, where such use is in the public interest
and is compatible with fair use”, shall not
constitute copyright infringement.
Intellectual Property Law ►►| 13

(c) With knowledge of infringing activity,


Filipino Society of Composers v. induces, causes or materially
Benjamin Tan contributes to the infringing conduct
The word “perform” as used in the Act has of another.
been applied to “One who plays a musical
composition on a piano, thereby producing Vicarious liability (benefited)
in the air sound waves w/c are heard as
music… and if the instrument he plays on is Contributory infringement liability (provided
a piano plus a broadcasting apparatus so the environment/setting for infringement)
that waves are thrown out, not only upon
the air, but upon the other, then also he is Primary infringement liability – (a)
performing the musical composition.” Secondary infringement liability – (b) and (c)

The playing of music in dine and dance IP Code as amended, Section 216.1
establishment which was paid for by the Remedies for Infringement –
public in purchases of food and drink Any person infringing a right protected
constituted “performance for profit” within a under this law shall be liable: x x x
Copyright Law.
(b) To pay to the copyright proprietor or
Philippine Education Co. v. Sotto his assigns or heirs such actual
Analyzing the language used, such news damages, including legal costs and other
items, editorial paragraphs and articles in expenses, as he may have incurred due
periodicals may be reproduced unless they to the infringement as well as the profits
contain a notice that their publication is the infringer may have made due to such
reserved or unless they contain a notice of infringement, and in proving profits the
copyright. In either event, the law specifically plaintiff shall be required to prove sales
provides that the source of the reproduction or only and the defendant shall be required
original reproduced shall be cited. to prove every element of cost which he
claims, or, in lieu of actual damages and
NBI-Microsoft Corp. v. Judy Hwang profits, such damages which to the court
These circumstances give rise to the shall appear to be just and shall not be
reasonable inference that respondents regarded as penalty: Provided, That the
mass-produced the CD-ROMs in question amount of damages to be awarded shall
without securing Microsoft’s prior be doubled against any person who:
authorization. (i) Circumvents effective
technological measures; or
(ii) Having reasonable grounds to
INFRINGEMENT AND DEFENSES know that it will induce, enable,
facilitate or conceal the
IP Code as amended, Section 216 infringement, remove or alter any
Infringement – electronic rights management
A person infringes a right protected under this information from a copy of a work,
Act when one: sound recording, or fixation of a
(a) Directly commits an infringement; performance, or distribute, import
(b) Benefits from the infringing activity for distribution, broadcast, or
of another person who commits an communicate to the public works
infringement if the person benefiting or copies of works without
has been given notice of the authority, knowing that electronic
infringing activity and has the right rights management information
and ability to control the activities of has been removed or altered
the other person; without authority.
Intellectual Property Law ►►| 14

xxx The effect of this provision is to facilitate in


the enforcement of copyrights especially in
In cases of circumventions of Technological cases where damages is not quantifiable.
Protection Measure and Rights Management
(TPMRM), the amount of damages shall be
doubled. (Double damages). IP Code, Section 217
Criminal Penalties –
The circumventions are treated as Section 217. 1 Any person infringing any
aggravating circumstances. right secured by provisions of Part IV of
this Act or aiding or abetting such
infringement shall be guilty of a crime
IP Code as amended, Sec.171.12 punishable by:
Technological measure means any (a) Imprisonment of one (1) year
technology, device or component that, in
to three (3) years plus a fine
the normal course of its operation,
ranging from Fifty thousand
restricts acts in respect of a work,
performance or sound recording, which pesos (P50,000) to One hundred
are not authorized by the authors, fifty thousand pesos (P150,000)
performers or producers of sound for the first offense.
recordings concerned or permitted by law
(b) Imprisonment of three (3)
years and one (1) day to six (6)
IP Code as amended, Sec.171.13
Rights management information means years plus a fine ranging from
information which identifies the work, One hundred fifty thousand
sound recording or performance; the pesos (P150,000) to Five hundred
author of the work, producer of the sound thousand pesos (P500,000) for
recording or performer of the the second offense.
performance; the owner of any right in
the work, sound recording or (c) Imprisonment of six (6) years
performance; or information about the and one (1) day to nine (9) years
terms and conditions of the use of the plus a fine ranging from Five
work, sound recording or performance; hundred thousand pesos
and any number or code that represent (P500,000) to One million five
such information, when any of these hundred thousand pesos
items is attached to a copy of the work, (P1,500,000) for the third and
sound recording or fixation of subsequent offenses.
performance or appears in conjunction
with the communication to the public of a (d) In all cases, subsidiary
work, sound recording or performance. imprisonment in cases of
insolvency.
IP Code as amended, Section 216.1
xxx 217.2. In determining the number of years
The copyright owner may elect, at any of imprisonment and the amount of fine,
time before final judgment is rendered, to the court shall consider the value of the
recover instead of actual damages and infringing materials that the defendant
profits, an award of statutory damages has produced or manufactured and the
for all infringements involved in an action damage that the copyright owner has
in a sum equivalent to the filing fee of the suffered by reason of the infringement.
infringement action but not less than Fifty
thousand pesos (Php50,000.00). 217.3. Any person who at the time when
copyright subsists in a work has in his
Intellectual Property Law ►►| 15

possession an article which he knows, or matter to which the action relates if


ought to know, to be an infringing copy of the defendant does not put in issue
the work for the purpose of: the question whether copyright
(a) Selling, letting for hire, or by subsists in the work or other subject
way of trade offering or exposing matter; and
for sale, or hire, the article; (b) Where the subsistence of the
(b) Distributing the article for copyright is established, the plaintiff
purpose of trade, or for any other shall be presumed to be the owner of
purpose to an extent that will the copyright if he claims to be the
prejudice the rights of the owner of the copyright and the
copyright owner in the work; or defendant does not put in issue the
(c) Trade exhibit of the article in question of his ownership.
public, shall be guilty of an (c) Where the defendant, without good
offense and shall be liable on faith, puts in issue the questions of
conviction to imprisonment and whether copyright subsists in a work
fine as above mentioned. or other subject matter to which the
action relates, or the ownership of
Damages copyright in such work or subject
No damages may be recovered under this matter, thereby occasioning
Act after 4 years from the time the cause of unnecessary costs or delay in the
action arose. proceedings, the court may direct that
any costs to the defendant in respect
of the action shall not be allowed by
him and that any costs occasioned by
IP Code, as amended, Section 218 the defendant to other parties shall be
Affidavit Evidence – paid by him to such other parties.
218.1. In an action under this Chapter, an
affidavit made before a notary public by Anton Piller Order
or on behalf of the owner of the copyright In the nature of civil search warrants in
in any work or other subject matter and order to prevent the removal or destruction
stating that: of evidence in copyright infringement cases.
(a) At the time specified therein,
copyright subsisted in the work or Rule on Search and Seizure in Civil
other subject matter; Actions for Infringement of IPR (A.M. No.
(b) He or the person named therein is the 02-1-06-SC)
owner of the copyright; and This is our own version of the Anton Piller
(c) The copy of the work or other subject order.
matter annexed thereto is a true copy
thereof. Section 1
The affidavit shall be admitted in Coverage –
evidence in any proceedings under this This Rule shall govern the provisional
Chapter and shall be prima facie proof of seizure and impounding of documents and
the matters therein stated until the articles in pending and intended civil
contrary is proved, and the court before actions for the purpose of preventing
which such affidavit is produced shall infringement and preserving relevant
assume that the affidavit was made by or evidence in regard to alleged infringement
on behalf of the owner of the copyright under Republic Act No. 8293, otherwise
known as the Intellectual property Code of
218.2. In an action under this Chapter. the Philippines, Article 50 of the Agreement
(a) Copyright shall be presumed to on Trade Related Aspects of intellectual
subsist in the work or other subject Property Rights, otherwise
Intellectual Property Law ►►| 16

known as TRIPS and other related laws (a) give impartial advise to the
and international conventions. alleged infringing defendant,
expected adverse party or to the
Section 6 person in charge of the premises
Grounds for the issuance of the order – to be searched as to the meaning
Before the Order can be issued, the and coverage of the writ;
evidence proffered by the applicant and (b) attempt to achieve agreement on
personally evaluated by the judge must a suitable search procedure;
show that: (c) assess what documents or
(a) the applicant is the right holder or articles come within the terms of
his duly authorized the writ;
representative; (d) ensure the accuracy of the list of
(b) there is probable cause to believe documents and articles searched,
that the applicant's right is being inspected, copied or seized by the
infringed or that such sheriff;
infringement is imminent and (e) prepare his own report on the
there is a prima facie case for final search and seizure and verify and
relief against the alleged sign the return prepared by the
infringing defendant or expected sheriff; and
adverse party; (f) generally, assist in the proper
(c) damage, potential or actual, likely execution of the writ.
to be caused to the applicant is The Commissioner shall be a member of
irreparable; the Philippine Bar and of proven
(d) there is demonstrable risk of competence, integrity and probity. He
evidence that the alleged shall receive such reasonable
infringing defendant or expected compensation as may be determined by
adverse party may destroy, hide the court which can be charged as cost of
or remove the documents or suit.
articles before any application
inter partes can be made; and
(e) the documents and articles to be
seized constitute evidence of the Section 20
alleged infringing defendant's or Failure to file Complaint -
expected adverse party's The writ shall also, upon motion of the
infringing activity or that they expected adverse party, be set aside and
infringe upon the intellectual the seized documents and articles
property right of the applicant or returned to the expected adverse party if
that they are used or intended to no case is filed with the appropriate court
be used as means of infringing or authority within thirty-one (31) calendar
the applicant's intellectual days from the date of issuance of the writ.
property right.
Section 21
Section 12 Claims for damages -
Commissioner, duties, qualifications, and Where the writ is discharged on
fees - any of the grounds provided in this Rule,
The enforcement of the writ shall be or where it is found after trial that there
supervised by the independent has been no infringement or threat of
Commissioner appointed by the court. In infringement of an intellectual property
the performance of his duty, the right, the court. Upon motion of the
Commissioner shall: alleged infringing defendant or expected
adverse party and after due hearing, shall
Intellectual Property Law ►►| 17

order the applicant to compensate the to promote the universal use of electronic
defendant or expected adverse party transactions in the government and by
upon the cash bond, surety bond or other the general public.
equivalent security for any injury or
damage the latter suffered by the Section 4
issuance and enforcement of the writ. Sphere of Application –
Should the damages exceed the amount This Act shall apply to any kind of
of the bond, the applicant shall be liable electronic document used in the context
for the payment of the excess. of commercial and non-commercial
When a complaint is already filed activities to include domestic and
in court, the motion shall be filed with the international dealings, transactions,
same court during the trial or before arrangements, agreements contracts and
appeal is perfected or before judgment exchanges and storage of information.
becomes executory, with due notice to
the applicant, setting forth the facts Section 33(b)
showing the defendant's right to Penalties –
damage's and the amount thereof. The Piracy or the unauthorized copying,
award of damages shall be included in reproduction, dissemination, or
the judgment in the main case. distribution, importation, use, removal,
Where no complaint is filed alteration, substitution, modification,
against the expected adverse party, the storage, uploading, downloading,
motion shall be filed with the court which communication, making available to the
issued the writ. In such a case, the court public, or broadcasting of protected
shall set the motion for summary hearing material, electronic signature or
and immediately determine the expected copyrighted works including legally
adverse party's right to damages. protected sound recording or phonograms
A judgment in favor of the or information material on protected works,
applicant in its principal claim, should not through the use of telecommunication
necessarily bar the alleged infringing networks, such as, but not limited to, the
defendant from recovering damages internet, in a manner that infringes
where he suffered losses by reason of the intellectual property rights shall be
wrongful issuance or enforcement of the punished by a minimum fine of One
writ. Hundred Thousand pesos (P
The damages provided for in this 100,000.00) and a maximum
section shall be independent from the commensurate to the damage incurred
damages claimed by the defendant in his and a mandatory imprisonment of six (6)
counterclaim. months to three (3) years.

Electronic Commerce Act (RA 8792)


Section 3 EXPLOITATION OF RIGHTS
Objective –
Commercialization of Copyright
This Act aims to facilitate domestic and
(Assignment and Licensing)
international dealings, transactions,
arrangements agreements, contracts and Copyright may be assigned in whole or in
exchanges and storage of information part.
through the utilization of electronic,
optical and similar medium, mode IP Code, Section 181
instrumentality and technology to Material Object -
The copyright is distinct from the property
recognize the authenticity and reliability
in the material object subject to it.
of electronic data messages or electronic
documents related to such activities and Consequently, the transfer or assignment
Intellectual Property Law ►►| 18

of the copyright shall not itself constitute People v. Ramos


a transfer of the material object. Nor shall The Copyright Law provides a two-year
a transfer or assignment of the sole copy period of prescription for acts punishable
or of one or several copies of the work thereunder, hence the four-year period of
imply transfer or assignment of the prescription in Act No. 3326 is not
copyright. applicable for Copyright Law violations.

Serrano Laktaw v. Paglinawan


Recordation Where one in publishing a Spanish-Tagalog
IP Code, Section 182 dictionary has but copied the equivalents,
Filing of Assignment or License – definitions and different meanings given in
An assignment or exclusive license may another's Spanish-Tagalog dictionary,
be filed in duplicate with the National although making some additions of his own
Library upon payment of the prescribed and some unimportant changes in the
fee for registration in books and records examples to illustrate the meanings of the
kept for the purpose. Upon recording, a words, such as substituting "Tayabas" for
copy of the instrument shall be, returned "Bulacan" in the expression "Voy a Bulacan"
to the sender with a notation of the fact of (I am going to Bulacan), it is evident that he
record. Notice of the record shall be merely reproduced the dictionary of the
published in the IPO Gazette. other author in violation of the Law of
January 10, 1879, on Intellectual Property.

Collecting Societies Columbia Pictures Inc. v. CA


IP Code, Section 183 It is our considered view that the 20th Century
Designation of Society – Fox ruling cannot be retroactively applied to
The owners of copyright and related the instant case to justify the quashal of
rights or their heirs may designate a Search Warrant No. 87–053. Herein
society of artists, writers, composers and petitioners’ consistent position that the order
other right-holders to collectively manage of the lower court of September 5, 1988
their economic or moral rights on their denying therein defendant’s motion to lift the
behalf. For the said societies to enforce order of search warrant was properly issued,
the rights of their members, they shall there having been satisfactory compliance
first secure the necessary accreditation with the then prevailing standards under the
form the Intellectual Property Office. law for determination of probable cause, is
indeed well taken. The lower court could not
Note: Read Secs. 186-189 in the Codal possibly have expected more evidence from
petitioners in their application for a search
warrant other than what the law and
20th Century Fox v. CA jurisprudence, then existing and judicially
Essence of copyright infringement is the accepted, required with respect to the finding
similarity of at least substantial similarity of of probable cause.
the purported pirated works to the
copyrighted work. Applicant must present to
the court the copyrighted films to compare
them with the purchased evidence of the LAW ON TRADEMARKS AND
video tapes allegedly pirated to determine UNFAIR COMPETITION
whether the latter is an unauthorized
reproduction of the former. To confirm the existing Philippine Intellectual
property regime with the TRIPS Agreement,
the new IP Code was enacted in 1998.
Certain changes were introduced in the
Intellectual Property Law ►►| 19

trademark law system to give substance to Others


the country’s accession to the TRIPS 4. Maintenance of Civil and Criminal
Agreement. Actions for Infringement (IP Code,
Sections 155-159)
HISTORY 5. Recordal of the COR in the Bureau of
During the 19th century, when a consumer Customs (to refuse entry of infringing
wanted to purchase particular goods, he had goods)
to rely on the expertise of the storeowner for
advice as to the origin of the goods and
quality. In effect, it is the advice/expertise of INTERNATIONAL AGREEMENTS
the storeowner that sells the product. TRADEMARKS
The Trademark register was first introduced 1. Paris Convention on the Protection of
in the UK in 1875 – to help, among others, Industrial Property
the consumer in selecting or choosing the - Art. 6bis (protection of well-known
products he wants to purchase. This is in marks)
response to the growth in trade involving a 2. Madrid Protocol
variety of UK goods. - National treatment
- Priority date: date in filing with the
At present, it is said that reputation/goodwill WIPO also considered date of filing
of a trademark (the origin) that sells in the country of application
products to the consumer – rather than the
expertise of the shopkeeper.
COLLECTIVE MARKS
DEFINITION IP Code, Section 121.2
A "trademark" is defined under R.A. 166, "Collective mark" means any visible sign
the Trademark Law, as including "any word, designated as such in the application for
name, symbol, emblem, sign or device or registration and capable of distinguishing
any combination thereof adopted and used the origin or any other common
by a manufacturer or merchant to identify characteristic, including the quality of
his goods and distinguish them from those goods or services of different enterprises
manufactured, sold or dealt in by others." which use the sign under the control of the
(Mipuri v. Court of Appeals, 318 SCRA 516, registered owner of the collective mark.
1999).
Example: common mark on Swiss products
Mark
IP Code, Section 121.1 ACQUIRING A MARK
"Mark" means any visible sign capable of
distinguishing the goods (trademark) or IP Code, Section 122
services (service mark) of an enterprise How marks are acquired? -
and shall include a stamped or marked The rights in a mark shall be acquired
container of goods. through a registration made validly in
accordance with the provisions of the law.

Can you use a mark before –


METHODS OF PROTECTION - The application?
- The registration?
1. Registration YES to both! All you need to present is
2. Opposition intent to use.
3. Cancellation
Intellectual Property Law ►►| 20

Note: Philippines, during the life of his


IP Code, Section 124.2 widow, if any, except by written
The applicant or the registrant shall file a consent of the widow;
declaration of actual use of the mark with (d) Is identical with a registered mark
evidence to that effect, as prescribed by belonging to a different proprietor or
the Regulations within three (3) years a mark with an earlier filing or priority
from the filing date of the application. date, in respect of:
Otherwise, the application shall be (i) The same goods or services, or
refused or the mark shall be removed (ii) Closely related goods or
from the Register by the Director. services, or
(iii) If it nearly resembles such a
mark as to be likely to deceive
FUNCTION or cause confusion;

a. Indicate origin
b. Guarantee quality Q: In Dermaline Inc v. Myra
c. Advertising Pharmaceutical Inc., were the competing
d. Investment marks used in closely related goods?
A: Yes. The competing trademarks of
Dermaline and Myra were used on closely
REGISTRABILITY OF MARKS related goods. In this case the Supreme
Court held that even though the products of
Examination of the trademark application
Dermaline and Myra belong to different
based on absolute and relative grounds:
classifications in their application for
Absolute
trademark registration before the IPO, the
Tests: Does it meet the definition of
purchasing public may still associate Myra
a mark? Is it distinctive?
with Dermaline especially considering that
Relative
both classifications pertain to treatments
Test: Is it identical or confusingly
for the skin.
similar to another mark?

IP Code, Section 123 Dermaline, Inc. v. Myra Pharmaceutical


Registrability – Inc.
123.1. A mark cannot be registered if it: Thus, the public may mistakenly think that
(a) Consists of immoral, deceptive or Dermaline is connected to or associated
scandalous matter, or matter which with Myra, such that, considering the current
may disparage or falsely suggest a proliferation of health and beauty products
connection with persons, living or in the market, the purchasers would likely
dead, institutions, beliefs, or national be misled that Myra has already expanded
symbols, or bring them into contempt its business through Dermaline from merely
or disrepute; carrying pharmaceutical topical applications
(b) Consists of the flag or coat of arms for the skin to health and beauty services.
or other insignia of the Philippines or
any of its political subdivisions, or of Q: In Birkenstock v. Phil Shoe Expo
any foreign nation, or any simulation Marketing Corporation, why were the
thereof; registrations of petitioner Birkenstock
(c) Consists of a name, portrait or approved despite the earlier registration
signature identifying a particular living issued to respondent Philippine Shoe
individual except by his written Expo for the same identical mark?
consent, or the name, signature, or A: The registrations of petitioner Birkenstock
portrait of a deceased President of the were approved despite the earlier
Intellectual Property Law ►►| 21

registration issued to Phil Shoe Expo for the whatsoever over the subject mark as
same identical mark because of two reasons: Cointreau is the true and lawful owner
First, Phil Shoe Expo failed to file its tenth thereof. As such, the IPO Director
year DAU (Declaration of Actual Use) of the General and the CA were correct in
mark resulting in its automatic cancellation by declaring Cointreau as the true and
virtue of Sec. 12 RA 166. In effect, Phil Shoe lawful owner of the subject mark and as
Expo abandoned all its rights over the mark.
such, is entitled to have the same
Second, and most important, is that
registered under its name.
Birkenstock was able to establish its true and
lawful ownership over the mark
“BIRKENSTOCK”. The Court stated that to Section 123.1
be an owner, Section 2-A of RA 166 does (f) Is identical with, or confusingly similar
not require that the actual use of a to, or constitutes a translation of a mark
trademark must be within the Philippines. considered well-known in accordance
with the preceding paragraph, which is
IP Code, Section 123.1
registered in the Philippines with respect
(e) Is identical with, or confusingly similar
to goods or services which are not
to, or constitutes a translation of a mark
similar to those with respect to which
which is considered by the competent
registration is applied for: Provided, That
authority of the Philippines to be well-
use of the mark in relation to those goods
known internationally and in the
or services would indicate a connection
Philippines, whether or not it is
between those goods or services, and the
registered here, as being already the
owner of the registered mark: Provided
mark of a person other than the
further, That the interests of the owner of
applicant for registration, and used for
the registered mark are likely to be
identical or similar goods or services:
damaged by such use;
Provided, That in determining whether a
mark is well-known, account shall be
taken of the knowledge of the relevant Q: In 246 Corporation v. Daway re: Roles
sector of the public, rather than of the and Rolex Music Lounge: is determining
public at large, including knowledge in infringement a question of fact?
the Philippines which has been
obtained as a result of the promotion of 246 Corporation v. Daway
the mark; In the case at bar, the Court of Appeals did
not err in finding that no abuse of discretion
could be ascribed to the trial courts denial of
Q: In the case of Ecole De Cuisine v. petitioner’s motion for preliminary hearing
Renaud Cointreau, Is Le Cordon Bleu a on its affirmative defenses with motion to
well-known mark in the Philippines? dismiss. The issue of whether or not a
trademark infringement exists, is a
Ecole De Cuisine Manille v. Renaud question of fact that could best be
Cointreau determined by the trial court.
It is thus clear that at the time Ecole started
using the subject mark, the same was
IP Code, Section 123.1 (g)
already being used by Cointreau, albeit
Is likely to mislead the public, particularly
abroad, of which Ecole’s directress was fully
as to the nature, quality, characteristics
aware, being an alumna of the latter’s or geographical origin of the goods or
culinary school in Paris, France. Hence, services;
Ecole cannot claim any tinge of ownership
Intellectual Property Law ►►| 22

commercial use of a mark in the


Q: In Shang Properties v. St. Francis, re: Philippines; (b) such use must result in
“St. Francis”, is the mark a geographical the distinctiveness of the mark insofar
name hence not capable of registration? as the goods or the products Shang
Properties Realty Corporation are
Shang Properties v. St. Francis concerned; and (c) proof of substantially
What is the criteria for determining whether exclusive and continuous commercial use in
a mark describes “geographical origin” of the Philippines for five (5) years before the
the goods/services, hence not registrable? date on which the claim of distinctiveness is
“…In order to determine whether or not the made.
geographic term in question is descriptively
used, the following question is relevant: (1) In this case, records would reveal that while
Is the mark the name of the place or it is true that respondent had been using the
region from which the goods actually mark “ST. FRANCIS” since 1992, its use
come? If the answer is yes, then the thereof has been merely confined to its
geographic term is probably used in a realty projects within the Ortigas Center, as
descriptive sense, and secondary specifically mentioned. As its use of the
meaning is required for protection.” mark is clearly limited to a certain locality, it
cannot be said that there was substantial
Shang Properties v. St. Francis commercial use of the same recognized all
In the US case of Great Southern Bank v. throughout the country. Neither is there any
First Southern Bank, descriptive showing of a mental recognition in buyers’
geographical terms are in the ‘public and potential buyers’ minds that products
domain’ in the sense that every seller connected with the mark “ST. FRANCIS”
should have the right to inform customers of are associated with the same source — that
the geographical origin of his goods. A is, the enterprise of respondent. Thus,
‘geographically descriptive term’ is any noun absent any showing that there exists a clear
or adjective that designates geographical goods/service-association between the
location and would tend to be regarded by realty projects located in the aforesaid area
buyers as descriptive of the geographic and herein respondent as the developer
location of origin of the goods or services. A thereof, the latter cannot be said to have
geographically descriptive term can indicate acquired a secondary meaning as to its use
any geographic location on earth, such as of the “ST. FRANCIS” mark.
continents, nations, regions, states, cities,
streets and addresses, areas of cities,
rivers, and any other location referred to by IP Code, Section 123.1
a recognized name. (h) Consists exclusively of signs that are
generic for the goods or services that
The word mark “St. Francis” is a geographic they seek to identify;
name that is not capable of registration by
express provision of Section 123.1(j) of the IP (i) Consists exclusively of signs or of
Code. St. Francis Development Corp. could indications that have become customary
only have acquired exclusive right to the word or usual to designate the goods or
mark “St. Francis” if it was proven that its use services in everyday language or in bona
of the mark has acquired secondary meaning. fide and established trade practice;
Under Section 123.2 specific requirements
have to be met in order to conclude that a (j) Consists exclusively of signs or of
geographically-descriptive mark has acquired indications that may serve in trade to
secondary meaning, to wit: (a) the secondary designate the kind, quality, quantity,
meaning must have arisen as a result of intended purpose, value, geographical
substantial origin, time or production of the goods or
Intellectual Property Law ►►| 23

rendering of the services, or other a conclusion as to the nature of the goods.


characteristics of the goods or services; Suggestive terms are eligible for trademark
protection under the law.
Q: In Society Des Produits Nestle v. CA,
re Master, if neither a generic nor
descriptive, what kind of mark is it? IP Code, Section 123.1
A: "MASTER" is a suggestive term brought (k) Consists of shapes that may be
about by the advertising scheme of Nestle. necessitated by technical factors or by
Suggestive terms are those which, in the the nature of the goods themselves or
phraseology of one court, require factors that affect their intrinsic value;
"imagination, thought and perception to
reach a conclusion as to the nature of the (l) Consists of color alone, unless defined
goods." Such terms, "which subtly connote by a given form; or
something about the product," are eligible
for protection in the absence of secondary Doctrine of Secondary Meaning
meaning. While suggestive marks are It only applies to (j)Descriptive; (k)Shapes;
capable of shedding "some light" upon (l) Color;
certain characteristics of the goods or
services in dispute, they nevertheless Note: Lyceum v. CA, re: Lyceum
involve "an element of incongruity," Doctrine of Secondary meaning is a word of
"figurativeness," or " imaginative effort on phrase originally incapable of exclusive
the part of the observer." appropriation, might nevertheless have been
See: IP Code, Section 123.1(j) used so long and so exclusively by one
producer with reference to his article that, in
Societe Des Produits Nestle v. CA trade and to that branch of the purchasing
Q: What trademark function was public, the word or phrase has come to mean
illustrated in the case? that the article was his product.
A: The trademark function illustrated in this
case is that trademarks advertise the Lyceum of the Philippines has not gained
products they symbolize. This can be exclusive use of “Lyceum” by long passage
gleaned from the fact that Robert Jaworski of time. The number alone of the private
and Atty. Ric Puno, Jr., the personalities respondents suggests strongly that the use
engaged to promote the product, are given of Lyceum has not been attended with the
the titles Master of the Game and Master of exclusivity essential for the applicability of
the Talk Show, respectively. In due time, the doctrine. It may be noted that one of the
because of these advertising schemes the respondents – Western Pangasinan
mind of the buying public had come to learn Lyceum used such term 17 years before the
to associate the word MASTER with the petitioner registered with the SEC.
opposer’s goods. Moreover, there may be other schools using
the name but not registered with the SEC
Q: Did respondent infringe a valid mark? because they have not adopted the
A: There is still an infringement of the mark corporate form of organization.
MASTER since the use of the same or similar
mark is likely to cause mistake or confusion or Ang v. Teodoro
even deceive ordinary purchasers. MASTER Teodoro established a shoe and slipper
is not an invalid trademark or un-registrable manufacturing business using the trademark
mark under Sec. 123.1 (j) of the IP Code since and tradename “Ang Tibay”. The business
it does not designate the characteristics of the became successful and “Ang Tibay” became
goods. MASTER is merely a suggestive term famous throughout the country. Ang
which still require imagination, thought and registered the trademark “Ang Tibay” for her
perception to reach pants and shirts business. One year later
Intellectual Property Law ►►| 24

Teodoro registered “Ang Tibay” as trademark.


Ang filed a complaint. Court held that “Ang
Tibay” is not a descriptive word and that even
WELL-KNOWN MARK
granting that it was a descriptive word, La Chemise Lacoste v. Fernandez
Teodoro’s long and exclusive use of the mark Pursuant to Treaty obligation, the Ministry of
entitled him to registration under the doctrine Trade directed the Director of the Patents
of secondary meaning. Office directing the latter: xxx xxx xxx

Q: Why was it not necessary for the ... to reject all pending applications for
Court to discuss the concept of Philippine registration of signature and other
secondary meaning? world famous trademarks by applicants
A: It was not necessary for the Court to other than its original owners or users.
discuss the concept of secondary meaning The conflicting claims over internationally
because it has already ruled that the phrase known trademarks involve such name
“Ang Tibay” is not a descriptive word within brands as Lacoste, Jordache, Gloria
the meaning of the Trademark Law. Hence Vanderbilt, Sasson, Fila, Pierre Cardin,
it could be registered as trademark. The Gucci, Christian Dior, Oscar de la Renta,
Doctrine of Secondary Meaning only applies Calvin Klein, Givenchy, Ralph Lauren,
when the subject mark is descriptive. Geoffrey Beene, Lanvin and Ted Lapidus.
… where warranted, Philippine registrants
The Doctrine of Secondary Meaning is to of such trademarks should be asked to
the effect that a word or phrase originally surrender their certificates of registration, if
incapable of exclusive appropriation any, to avoid suits for damages and other
because geographically or otherwise legal action by the trademarks' foreign or
descriptive, might nevertheless be local owners or original users.
registered because of long and exclusive
use of said phrase with reference to TEST TO DETERMINE CONFUSING
products and business as to acquire a SIMILARITY
proprietary connotation.
1. Dominancy Test
Maintenance The Dominancy Test focuses on the
IP Code, Section 123.2 similarity of the prevalent features of the
As regards signs or devices mentioned in competing trademarks that might cause
paragraphs (j), (k), and (l), nothing shall confusion or deception.
prevent the registration of any such sign 2. Totality or Holistic Test
or device which has become distinctive in Holistic Test entails a consideration of the
relation to the goods for which entirety of the marks as applied to the
registration is requested as a result of the products, including labels and packaging,
use that have been made of it in in determining confusing similarity. The
scrutinizing eye of the observer must
commerce in the Philippines. The Office
focus not only on the predominant words
may accept as prima facie evidence that
but also on the other features appearing in
the mark has become distinctive, as used both labels so that a conclusion may be
in connection with the applicant's goods drawn as to whether one is confusingly
or services in commerce, proof of similar to the other.
substantially exclusive and continuous
use thereof by the applicant in commerce
in the Philippines for five (5) years before
the date on which the claim of
distinctiveness is made.
Intellectual Property Law ►►| 25

DOMINANCY TEST v. HOLISTIC TEST prescription. The common trade channel is


the pharmacy or the drugstore.
Etepha v. Directory of Patents
Westmont Pharmaceuticals sought the Note: In this case, marks “ATUSSIN” and
registration of its product ATUSSIN. Etepha “PERTUSSIN” were not said to be similar.
opposed the application for registration on the Their labels were written differently. As
ground that it is confusingly similar to its they appear on the respective labels,
product PERTUSSIN which has already been these words are presented to the public
registered and also serves as a treatment for in different styles of writing and methods
cough. Court held that by going into the whole of design. The horizontal plain, block
letters of Atussin and the diagonally and
of the two marks, it could hardly be said that
artistically upward writing of Pertussin
they are similar. The appearance of the labels
leave distinct visual impressions.
were different, and the marks do not sound
They do not sound alike. There is not
alike. Moreover, it has been a practice by much phonetic similarity between the
pharma companies to use words suggestive two.
of the ailments they treat in naming their
products. TUSSIS is the Latin term for cough.
To add, the Court also held that the possibility Emerald Garment Manufacturing v. CA
of confusion is even made more remote H.D. Lee & Co. is a US Corp which uses the
because it is the practice of intending buyers trademark “LEE” on its products such as pants
to first secure a prescription from the doctor. and jeans. Emerald Garment is a domestic
corporation which is also engaged in the
manufacture and selling of pants and jeans
Q: What test was used in the case? using the trademark “STYLISTIC MR. LEE”.
The court used the Holistic Test. A practical H.D. Lee applied for the cancellation of the
approach to the problem of similarity or trademark registration of “STYLISTIC MR.
dissimilarity is to go into the whole of the LEE” on the ground that it is confusingly
two trademarks pictured in their manner of similar to its trademark “LEE”. The Court held
display. The trademark complained of that the two trademarks are dissimilar viewed
should be compared and contrasted with as a whole and by considering that consumers
the purchaser’s memory (not in are less likely to be deceived or misled by
juxtaposition) of the trademark said to be either of the two marks. The Court also noted
infringed. Some such factors as “sound; that under RA 166 a personal name or
appearance; form, style, shape, size or surname is not a registrable mark. Hence the
format; color; ideas connoted by marks; the action for infringement against Emerald
meaning, spelling, and pronunciation of Garment must necessarily fail.
words used and the setting in which the
words appears” may be considered.
Explain how the Court resolved the case
in “two ways.”
Q: Identify a distinctive consumer
1. First, by determining whether there is
practice when purchasing medicines.
colorable imitation of the mark “LEE” in
An intending buyer must have to go first to a
the subsequent mark “STYLISTIC
licensed doctor of medicine; he receives
MR.LEE”.
instructions as to what to purchase; he reads
In determining whether colorable
the doctor’s prescription; he knows what he is
imitation exists, jurisprudence has
to buy. He is not of the incautious, unwary,
developed two kinds of tests—the
unobservant or unsuspecting type; he
Dominancy Test applied in Asia
examines the product sold to him; he checks
Brewery, Inc. v. Court of Appeals and
to find out whether it conforms to the medical
other cases and the Holistic Test
Intellectual Property Law ►►| 26

developed in Del Monte Corporation with its SAN MIGUEL PALE PILSEN. SC
v. Court of Appeals and its RULING: (applying the dominancy test) The
proponent cases. trial court perceptively observed that the word
Petitioner’s trademark is the whole “BEER” does not appear in SMC’s trademark,
“STYLISTIC MR. LEE.” Although on just as the words “SAN MIGUEL” do not
its label the word “LEE” is prominent, appear in ABI’s trademark. Hence, there is
the trademark should be considered absolutely no similarity in the dominant
as a whole and not piecemeal. The features of both trademarks.
dissimilarities between the two marks
become conspicuous, noticeable and Neither in sound, spelling or appearance
substantial enough to matter can BEER PALE PILSEN be said to be
especially in the light of the following confusingly similar to SAN MIGUEL PALE
variables that must be factored in. PILSEN. There are other dissimilarities like
Purchaser is more cautious and bottle appearance, slogan and substantial
discrimination in their purchase of price difference. The fact that the words
maong pants because they are pale pilsen are part of ABI’s trademark does
expensive. not constitute an infringement of SMC’s
The average Filipino consumer buys trademark: SAN MIGUEL PALE PILSEN, for
his jeans by brand (just like in “pale pilsen” are generic words descriptive
purchase of beers). of the color (“pale”), of a type of beer
The test of fraudulent simulation is (“pilsen”), which is a light bohemian beer
to be found in the likelihood of the with a strong hops flavor that originated in
deception of some persons in some the City of Pilsen in Czechoslovakia and
measure acquainted with an became famous in the Middle Ages.
established design and desirous of HENCE NO INFRINGEMENT
purchasing the commodity with which
that design has been associated. The When as in this case, the names of the
simulation, in order to be competing products are clearly different and
objectionable, must be such as their respective sources are prominently
appears likely to mislead the printed on the label and on other parts of the
ordinary intelligent buyer who has a bottle, mere similarity in the shape and size of
need to supply and is familiar with the the container and label, does not constitute
article that he seeks to purchase. unfair competition. Stenie bottle is a standard
for beer and universally used. No proof on
record that BEER PALE PILSEN is being
2. Second, by considering what marks passed off as SAN MIGUEL PALE
could be and could not be registered. PILSEN. HENCE NO UNFAIR
A personal name or surname may not be COMPETITION.
monopolized as a trademark or tradename
as against others of the same name or What is trade dress?
surname under Sec. 4 of R.A. A trade dress refers to the characteristics of
No.166. “LEE” is primarily a surname. the general appearance of a product.
Private respondent cannot, therefore,
acquire exclusive ownership over and Del Monte Corporation v. CA
singular use of said term. Del Monte is a foreign corp not registered in
the PH but it has authorized PhilPack as its
Asia Brewery v. CA representative to apply for trademark
San Miguel Corp (SMC) filed a case for registration of its name DEL MONTE and its
infringement and unfair competition against bottle configuration. Among the products in
Asia Brewery Inc (ABI) in connection with the the case is catsup PhilPack registered, in
ABI’s BEER PALE PILSEN which competes behalf of Del Monte, the bottle configuration
Intellectual Property Law ►►| 27

and the mark DEL MONTE. Sunshine Sauce undergarments. Respondent General
Manufacturing is a domestic corp which sells Garments was the registrant of FRUIT OF
various sauces like the Sunshine fruit catsup. EVE mark also dealing with women’s
It was discovered that Sunshine uses the panties, pajamas etc. Fruit of the Loom Inc.
bottles of Del Monte which it bought from junk filed an action for infringement and unfair
shops in its catsup business. Del Monte and competition against General Garments
PhilPack sued for infringement and unfair claiming that the two marks are confusingly
competition. The court held that there was similar The Court held that there was no
infringement and unfair competition. There confusing similarity between the two there
was infringement because of the similarities in were in fact glairing dissimilarities as to
the marks. There was unfair competition appearance to the extent that the purchaser
because Sunshine used the bottles of DEL would not be likely to be confused with the
MONTE even after the latter made a demand two products.
for the former to stop.
What is the dominant feature of the
What test was used in the case? competing marks?
The test used was the holistic test. To There was no common dominant features
determine whether a trademark has been between the competing marks. In this case
infringed, we must consider the mark as a the court ruled that the similar word FRUIT
whole and not as dissected. between the two marks does not even
constitute dominant feature. In the
What is the finding of the Court: trademarks FRUIT OF THE LOOM and
infringement or unfair competition? FRUIT FOR EVE, the lone similar word is
Sunshine’s label is an infringement of the FRUIT. WE agree with the respondent court
Del Monte’s trademark, law and equity call that by mere pronouncing the two marks, it
for the cancellation of the private could hardly be said that it will provoke a
respondent’s registration and withdrawal of confusion, as to mistake one for the other.
all its products bearing the questioned label Standing by itself, FRUIT OF THE LOOM is
from the market. wholly different from FRUIT FOR EVE. The
dominant feature of both trademarks is
NOT the word FRUIT for even in the
The predominant colors used in the Del printing of the trademark in both hang tags,
Monte label are green and red-orange, the the word FRUIT is not at all made dominant
same with Sunshine. The word “catsup” in over the other words.
both bottles is printed in white and the style
of the print/letter is the same. Although the What test was used in the case?
logo of Sunshine is not a tomato, the figure No express mention but probably the
nevertheless approximates that of a tomato. dominancy test because the Court tried to
determine the dominant features of the
With regard to the use of Del Monte’s bottle, marks
the same constitutes unfair competition;
hence, the respondent should be
permanently enjoined from the use of such Berris Agricultural Co., Inc. v. Norvy
Abyadang
bottles.
Abdayang applied for the registration of its
mark “NS D-10 PLUS” for use in connection
Fruit of the Loom v. CA with fungicide. Berris Agricultural opposed the
Fruit of the Loom Inc. is the registrant of the application for registration on the ground that
mark FRUIT OF THE LOOM mark and is it is confusingly similar to its registered mark
engaged in the business of selling “D-10 80 WP” for the same product and
Intellectual Property Law ►►| 28

the same active ingredient. The Court ruled schemes (red, green, and white); and the
that the two marks are confusingly similar marks are both predominantly red in color,
applying both the dominancy and holistic with the same phrase “BROAD SPECTRUM
tests. FUNGICIDE” written underneath.

How do you overcome prima facie


presumptions brought about by the
registration of a mark?
The prima facie presumption brought about by INTRODUCTION TO THE SYSTEM OF
the registration of a mark may be challenged
and overcome, in an appropriate action, by
REGISTRATION
proof of the nullity of the registration or of non- WHO MAY APPLY?
use of the mark, except when excused. Rules and Regulations on Trademarks, Rule 300
Moreover, the presumption may likewise be The Applicant –
defeated by evidence of prior use by another (a) Applicant may be a person or juridical
person, i.e., it will controvert a claim of legal person.
appropriation or of ownership based on (b) Unless modified by this Chapter, all
registration by a subsequent user. This is applications for a mark should be in the
name of the applicant(s) who may sign
because a trademark is a creation of use and
the application. If there are more than one
belongs to one who first used it in trade or
applicant, all of them should be named as
commerce. applicant but anyone may sign the
What test was used in the case? application for and in behalf of all the
Both dominancy test and holistic test. applicants.

Applying the Dominancy Test, it cannot be Rules and Regulations on Trademarks, Rule 303
gainsaid that Abyadang’s “NS D-10 PLUS” Applicant may be represented by attorney.
is similar to Berris’ “D-10 80 WP,” that —
confusion or mistake is more likely to occur. The owner of a mark may file and prosecute
Undeniably, both marks pertain to the same his own application for registration, or he
type of goods—fungicide with 80% may be represented by any attorney or
Mancozeb as an active ingredient and used other person authorized to practice in such
for the same group of fruits, crops, matters by the Office. The Office shall not
aid in the selection of an attorney or agent
vegetables, and ornamental plants, using
other than the furnishing of the list of
the same dosage and manner of
Attorneys or agents authorized to practice
application. They also belong to the same before the Office.
classification of goods under R.A. No. 8293.
Both depictions of “D-10,” as found in both Note: Prior use of a mark (upon trademark
marks, are similar in size, such that this filing) not required.
portion is what catches the eye of the
purchaser. Undeniably, the likelihood of Documents/Information Necessary to
confusion is present. Support a Philippine Trademark
Application:
1. Executed Trademark Application
This likelihood of confusion and mistake is
The applicant must indicate the names of
made more manifest when the Holistic
the goods or services for which the
Test is applied, taking into consideration the registration is sought, grouped according
packaging, for both use the same type of to the Nice Classification. There are
material (foil type) and have identical color
Intellectual Property Law ►►| 29

thirty-four (34) classes of goods and eight Applications before publication are
(8) classes of businesses and services. confidential.
2. Formal Drawing of the Mark;
3. Certified Copy of the Home/Foreign After Publication
Application (IP Code, Section 131) Publication is for opposition purposes.
NOTE: This document is required
only for applications claiming Public is given an opportunity to object the
Convention priority and must be filed application.
within six (6) months from date of Third party may commence an Opposition
earliest filing. (Rule 203) Proceeding (both parties to submit evidence
to support claims)

FOREIGN APPLICANTS After the proceedings, the application is


IP Code, Section 3 either approved or disapproved.
International Conventions and
Reciprocity. —
Any person who is a national or who is Trademark Application Procedure
domiciled or has a real and effective Overview
industrial establishment in a country
which is a party to any convention, treaty
or agreement relating to intellectual Filing of an
Application
property rights or the repression of unfair
competition, to which the Philippines is
also a party, or extends reciprocal rights
to nationals of the Philippines by law, Search &
Examination
shall be entitled to benefits to the extent
necessary to give effect to any provision
of such convention, treaty or reciprocal
Issuance of
law, in addition to the rights to which any Registrability
owner of an intellectual property right is Report
otherwise entitled by this Act

Response
PROCEDURE
PROCEDURE FOR REGISTRATION
(IP Code, Sections 127-132) Recommendation
Rejection
for Allowance
Prior to publication
Ex-Parte Proceedings or law contest
between the examiner (representing public
interest) and the applicant. Publication

The examiner determines whether the mark


is registrable (formal and substantive Issuance of the
requirements) Certificate of
Registration

If Yes, Director will commend to the Director


of the Bureau of Trademarks the publication Second
of the mark Publication
Intellectual Property Law ►►| 30

FILING DATE those in respect of which the trademark


IP Code, Section 127 is registered where such use would result
Filing Date – in a likelihood of confusion. In case of the
Section 127.1 use of an identical sign for identical
Requirements. — goods or services, a likelihood of
The filing date of an application shall be confusion shall be presumed.
the date on which the Office received the
following indications and elements in NOTE: Actual confusion is not necessary.
English or Filipino: DURATION
(a) An express or implicit indication that
the registration of a mark is sought; IP Code, Section 145
(b) The identity of the applicant; Duration –
(c) Indications sufficient to contact the A certificate of registration shall remain
applicant or his representative, if any; in force for ten (10) years: Provided, That
(d) A reproduction of the mark whose the registrant shall file a declaration of
registration is sought; and actual use and evidence to that effect, or
(e) The list of the goods or services for shall show valid reasons based on the
which the registration is sought. existence of obstacles to such use, as
prescribed by the Regulations, within one
127.2. No filing date shall be accorded until (1) year from the fifth anniversary of the
the required fee is paid. date of the registration of the mark.
Otherwise, the mark shall be removed
PRIORITY RIGHT from the Register by the Office.
IP Code, Section 131.2
No registration of a mark in the RENEWAL
Philippines by a person described in this IP Code, Section 146
section shall be granted until such mark Renewal. –
has been registered in the country of 146.1. A certificate of registration may be
origin of the applicant. renewed for periods of ten (10) years at its
expiration upon payment of the prescribed
CERTIFICATES OF REGISTRATION fee and upon filing of a request.
IP Code, Section 138
Certificates of Registration –
A certificate of registration of a mark shall COMMERCIALIZATION OF TRADE
be prima facie evidence of the validity of MARKS
the registration, the registrant's
ownership of the mark, and of the COMMERCIAL EXPLOITATION
registrant's exclusive right to use the IP Code, Section 149
same in connection with the goods or Assignment and Transfer of Application
services and those that are related and Registration. –
thereto specified in the certificate. 149.1. An application for registration of a
mark, or its registration, may be assigned
RIGHTS CONFERRED or transferred with or without the transfer
IP Code, Section 147.1 of the business using the mark.
Rights Conferred –
The owner of a registered mark shall have IP Code, Section 150
the exclusive right to prevent all third License Contracts. –
parties not having the owner's consent Any license contract concerning the
from using in the course of trade identical registration of a mark, or an application
or similar signs or containers for goods or therefor, shall provide for effective control
services which are identical or similar to by the licensor of the quality of the goods
Intellectual Property Law ►►| 31

or services of the licensee in connection justified in damaging or jeopardizing


with which the mark is used. If the license another's business by fraud, deceit, trickery
contract does not provide for such or unfair methods of any sort. This
quality control, or if such quality control necessarily precludes the trading by one
is not effectively carried out, the license dealer upon the good name and reputation
contract shall not be valid. built up by another (Baltimore v. Moses, 182
Md 229, 34 A (2d) 338).

BASIS OF REGISTRATION Birkenstock v. Phil Shoe Expo Marketing


Corp.
BASIS OF REGISTRATION Clearly, it is not the application or
1. Local Use (intent to use will suffice) registration of a trademark that vests
2. Foreign Application/Registration ownership thereof, but it is the ownership of
Applicant may prosecute application by a trademark that confers the right to register
himself or by an attorney (latter case, the same. A trademark is an industrial
Power of Attorney required) property over which its owner is entitled to
property rights which cannot be
Resident agent required for foreign appropriated by unscrupulous entities that,
applicant in one way or another, happen to register
such trademark ahead of its true and lawful
IMPORTANCE OF REGISTRATION owner. The presumption of ownership
The registration of marks secures to the accorded to a registrant must then
registrant the advantages which are denied necessarily yield to superior evidence of
to a non-registrant or ordinary user. actual and real ownership of a trademark.

While failure to register will not prevent the The SC recognized normal potential
owner from continuing its use, the law expansion of his business
deprives him of benefits which the law
grants to registrants of marks. Dermaline, Inc v. Myra Pharmaceutical
Inc.
Modern law recognizes that the protection to
which the owner of a trademark is entitled is
Ecole De Cuisine Manille v. Renaud not limited to guarding his goods or business
Cointreau from actual market competition with identical
Republic Act No. 8293, otherwise known as or similar products of the parties, but extends
the Intellectual Property Code of the to all cases in which the use by a junior
Philippines, as amended, has already appropriator of a trade-mark or trade-name is
dispensed with the requirement of prior actual likely to lead to a confusion of source, as
use at the time of registration. Thus, there is where prospective purchasers would be
more reason to allow the registration of the misled into thinking that the complaining
subject mark under the name of Cointreau as party has extended his business into the
its true and lawful owner. field or is in any way connected with the
activities of the infringer; or when it forestalls
La Chemise Lacoste v. Fernandez the normal potential expansion of his
The law on trademarks and tradenames is business
based on the principle of business integrity
and common justice' This law, both in letter Fredco Manufacturing Corp v. Pres. &
and spirit, is laid upon the premise that, Fellows of Harvard College
while it encourages fair trade in every way There are two compelling reasons why
and aims to foster, and not to hamper, Fredco’s petition must fail.
competition, no one, especially a trader, is
Intellectual Property Law ►►| 32

1. First, Fredcos registration of the mark DILUTION


Harvard and its identification of origin as
Cambridge, Massachusetts falsely
Trademark dilution is the lessening of the
suggest that Fredco or its goods are
capacity of a famous mark to identify and
connected with Harvard University;
distinguish goods or services, regardless of
2. Fredcos registration of the mark Harvard
the presence or absence of:
should not have been allowed because
1. Competition between the owner of the
Section 4(a) of R.A. No. 166 prohibits
famous mark and other parties; or
the registration of a mark which may
2. Likelihood of confusion, mistake or
disparage or falsely suggest a
deception.
connection with persons, living or dead,
institutions, beliefs x x x. Section 4(a)
of R.A. No. 166 Subject to the principles of equity, the owner
of a famous mark is entitled to an injunction
against another person’s commercial use in
Mipuri v. Court of Appeals the commerce of a mark or trade-name, if
In Philippine jurisprudence, the function of a such use begins after the mark has become
trademark is to point out distinctly the origin famous and causes dilution of the distinctive
or ownership of the goods to which it is quality of the mark.
affixed; to secure to him, who has been
instrumental in bringing into the market a OPPOSITION
superior article of merchandise, the fruit of
his industry and skill; to assure the public IP Code, Section 134
that they are procuring the genuine article; Opposition –
to prevent fraud and imposition; and to Any person who believes that he would
protect the manufacturer against be damaged by the registration of a mark
substitution and sale of an inferior and may, upon payment of the required fee
different article as his product. and within thirty (30) days after the
publication referred to in Subsection
Lyceum of the Philippines v. Court of 133.2, file with the Office an opposition to
Appeals the application. Such opposition shall be
The policy underlying the prohibition in in writing and verified by the oppositor or
Section 18 against the registration of a by any person on his behalf who knows
corporate name which is "identical or the facts, and shall specify the grounds
deceptively or confusingly similar" to that of on which it is based and include a
any existing corporation or which is statement of the facts relied upon. Copies
"patently deceptive" or "patently confusing" of certificates of registration of marks
or "contrary to existing laws," is the registered in other countries or other
avoidance of fraud upon the public which supporting documents mentioned in the
would have occasion to deal with the entity opposition shall be filed therewith,
concerned, the evasion of legal obligations together with the translation in English, if
and duties, and the reduction of difficulties not in the English language. For good
of administration and supervision over cause shown and upon payment of the
corporations. required surcharge, the time for filing an
opposition may be extended by the
Director of Legal Affairs, who shall notify
the applicant of such extension. The
Regulations shall fix the maximum period
of time within which to file the opposition.
Intellectual Property Law ►►| 33

Any person who believes that he would be A certificate of registration may be renewed.
damaged by the registration of the mark
may file an opposition to the application. Note: This is different from that which is
required under Sec.124.2 (DAU after
CANCELLATION/ REVOCATION application):
IP Code, Section 151.1 IP Code, Section 124.2
Cancellation – The applicant or the registrant shall file a
(a) Within five (5) years from the date of declaration of actual use of the mark with
the registration of the mark under evidence to that effect, as prescribed by
this Act. the Regulations within three (3) years
(b) At any time, if the registered mark from the filing date of the application.
becomes the generic name for the Otherwise, the application shall be
goods or services, or a portion thereof, refused or the mark shall be removed
for which it is registered, or has been from the Register by the Director.
abandoned, or its registration was
obtained fraudulently or contrary to the One time DAU (within three years from filing)
provisions of this Act, or if the
registered mark is being used by, or The submission of a DAU or Declaration of
with the permission of, the registrant Non-Use is required to maintain a trademark
so as to misrepresent the source of the application and registration. The acceptable
goods or services on or in connection grounds for non-use are restricted to:
with which the mark is used. xxx. (n)
- Prohibition of sale imposed by
government regulation;
(c) At any time, if the registered owner of
the mark without legitimate reason - Court-issued injunctions;
fails to use the mark within the - Pending application with the Food
Philippines, or to cause it to be used and Drugs;
in the Philippines by virtue of a - Administration; and
license during an uninterrupted - Pending litigation or opposition
period of three (3) years or longer. (IP Code, Section 124.2)

A petition to cancel a registration of a mark


may be filed with the Bureau of Legal Affairs TRADE NAMES OR BUSINESS NAMES
by any person who believes that he is or will IP Code, Section 165
be damaged by the registration of a mark. Trade Names or Business Names –
165.1. A name or designation may not be
- Generally within 5 years
used as a trade name if by its nature or
- At any time after the registration the use to which such name or
under circumstances in (b) and
designation may be put, it is contrary to
(c) public order or morals and if, in
particular, it is liable to deceive trade
MAINTENANCE circles or the public as to the nature of
the enterprise identified by that name.
See also Sec. 145 and 146.

Declaration of actual use and evidence to


that effect within 1 year from the 5th 165.2.
anniversary of the date of the registration. (a) Notwithstanding any laws or
regulations providing for any
Intellectual Property Law ►►| 34

obligation to register trade names, dominant feature thereof and apply such
such names shall be protected, even reproduction, counterfeit, copy or
prior to or without registration, colorable imitation to labels, signs,
against any unlawful act committed prints, packages, wrappers, receptacles
by third parties. or advertisements intended to be used in
(b) In particular, any subsequent use of commerce upon or in connection with the
the trade name by a third party,
sale, offering for sale, distribution, or
whether as a trade name or a mark or
advertising of goods or services on or in
collective mark, or any such use of a
similar trade name or mark, likely to connection with which such use is likely
mislead the public, shall be deemed to cause confusion, or to cause mistake,
unlawful. or to deceive, shall be liable in a civil
action for infringement by the registrant
165.3. The remedies provided for in for the remedies hereinafter set forth:
Sections 153 to 156 and Sections 166 and Provided, That the infringement takes
167 shall apply mutatis mutandis. place at the moment any of the acts
stated in Subsection 155.1 or this
165.4. Any change in the ownership of a subsection are committed regardless of
trade name shall be made with the whether there is actual sale of goods or
transfer of the enterprise or part thereof services using the infringing material.
identified by that name. The provisions of
Subsections 149.2 to 149.4 shall apply
mutatis mutandis. IP Code, Section 156
Actions, and Damages and Injunction for
Infringement. —
TRADEMARK INFRINGEMENT 156.1. The owner of a registered mark
may recover damages from any person
IP Code, Section 155 who infringes his rights, and the measure
Remedies; Infringement. – of the damages suffered shall be either
Any person who shall, without the consent the reasonable profit which the
of the owner of the registered mark: complaining party would have made, had
155.1. Use in commerce any reproduction, the defendant not infringed his rights, or
counterfeit, copy, or colorable imitation of a the profit which the defendant actually
registered mark or the same container or a made out of the infringement, or in the
dominant feature thereof in connection with event such measure of damages cannot
the sale, offering for sale, distribution, be readily ascertained with reasonable
advertising of any goods or services certainty, then the court may award as
including other preparatory steps damages a reasonable percentage based
necessary to carry out the sale of any upon the amount of gross sales of the
goods or services on or in connection with defendant or the value of the services in
which such use is likely to cause connection with which the mark or trade
confusion, or to cause mistake, or to name was used in the infringement of the
deceive; or rights of the complaining party.

Note: Dominancy test was expressly 156.2. On application of the complainant,


adopted in the above provision of the IP the court may impound during the
Code. pendency of the action, sales invoices
and other documents evidencing sales.
155.2. Reproduce, counterfeit, copy or
colorably imitate a registered mark or a
Intellectual Property Law ►►| 35

156.3. In cases where actual intent to within a circle or if the defendant had
mislead the public or to defraud the otherwise actual notice of the registration.
complainant is shown, in the discretion of
the court, the damages may be doubled.
IP Code, Section 159
156.4. The complainant, upon proper Limitations to Actions for Infringement.
showing, may also be granted injunction. — Notwithstanding any other provision of
this Act, the remedies given to the owner
IP Code, Section 157 of a right infringed under this Act shall be
Power of Court to Order Infringing limited as follows:
Material Destroyed – 159.1. Notwithstanding the
157.1. In any action arising under this provisions of Section 155 hereof,
Act, in which a violation of any right of a registered mark shall have no
the owner of the registered mark is effect against any person who, in
established, the court may order that good faith, before the filing date
goods found to be infringing be, without or the priority date, was using
compensation of any sort, disposed of the mark for the purposes of his
outside the channels of commerce in business or enterprise: Provided,
such a manner as to avoid any harm That his right may only be
caused to the right holder, or destroyed; transferred or assigned together
and all labels, signs, prints, packages, with his enterprise or business
wrappers, receptacles and or with that part of his enterprise
advertisements in the possession of the or business in which the mark is
defendant, bearing the registered mark or used.
trade name or any reproduction,
counterfeit, copy or colorable imitation 159.2. Where an infringer who is
thereof, all plates, molds, matrices and engaged solely in the business
other means of making the same, shall be of printing the mark or other
delivered up and destroyed. infringing materials for others is
an innocent infringer, the owner
157.2. In regard to counterfeit goods, the of the right infringed shall be
simple removal of the trademark affixed entitled as against such infringer
shall not be sufficient other than in only to an injunction against
exceptional cases which shall be future printing.
determined by the Regulations, to permit
the release of the goods into the 159.3. Where the infringement
channels of commerce. complained of is contained in or is
part of paid advertisement in a
IP Code, Section 158 newspaper, magazine, or other
Damages; Requirement of Notice. — similar periodical or in an
In any suit for infringement, the owner of electronic communication, the
the registered mark shall not be entitled to remedies of the owner of the right
recover profits or damages unless the acts infringed as against the publisher
have been committed with knowledge that or distributor of such newspaper,
such imitation is likely to cause confusion, magazine, or other similar
or to cause mistake, or to deceive. Such periodical or electronic
knowledge is presumed if the registrant communication shall be limited
gives notice that his mark is registered by to an injunction against the
displaying with the mark the words presentation of such advertising
'"Registered Mark" or the letter R matter in future issues of such
newspapers, magazines, or other
Intellectual Property Law ►►| 36

similar periodicals or in future services from those of others, whether or


transmissions of such electronic not a registered mark is employed, has a
communications. The limitations property right in the goodwill of the said
of this subparagraph shall apply goods, business or services so identified,
only to innocent infringers: which will be protected in the same
Provided, That such injunctive manner as other property rights.
relief shall not be available to the
owner of the right infringed with 168.2. Any person who shall employ
respect to an issue of a deception or any other means contrary to
newspaper, magazine, or other good faith by which he shall pass off the
similar periodical or an electronic goods manufactured by him or in which
communication containing he deals, or his business, or services for
infringing matter where restraining those of the one having established such
the dissemination of such goodwill, or who shall commit any acts
infringing matter in any particular calculated to produce said result, shall be
issue of such periodical guilty of unfair competition, and shall be
or in an electronic subject to an action therefor.
communication would delay the
delivery of such issue or 168.3. In particular, and without in any way
transmission of such electronic limiting the scope of protection against
communication is customarily unfair competition, the following shall be
conducted in accordance with deemed guilty of unfair competition:
the sound business practice, and (a) Any person, who is selling his goods
not due to any method or device and gives them the general
adopted to evade this section or appearance of goods of another
to prevent or delay the issuance manufacturer or dealer, either as to
of an injunction or restraining the goods themselves or in the
order with respect to such wrapping of the packages in which
infringing matter. they are contained, or the devices or
words thereon, or in any other
feature of their appearance, which
UNFAIR COMPETITION would be likely to influence
What is Unfair Competition? purchasers to believe that the goods
“Any act of competition contrary to honest offered are those of a manufacturer
practices in industrial and commercial or dealer, other than the actual
matters.” (Art.10bis, Paris Convention) manufacturer or dealer, or who
otherwise clothes the goods with
Q: Is unfair competition an intellectual such appearance as shall deceive the
property? public and defraud another of his
A: No. But the protection against unfair legitimate trade, or any subsequent
competition is recognized under IP laws. It vendor of such goods or any agent of
becomes an extension of trademark law. any vendor engaged in selling such
goods with a like purpose;
(b) Any person who by any artifice, or
device, or who employs any other
IP Code, Section 168 means calculated to induce the false
Unfair Competition, Rights, Regulation belief that such person is offering the
and Remedies. – services of another who has
168.1. A person who has identified in the identified such services in the mind
mind of the public the goods he of the public; or
manufactures or deals in, his business or
Intellectual Property Law ►►| 37

(c) Any person who shall make any false Jurisprudential Definition of Unfair
statement in the course of trade or Competition
who shall commit any other act
contrary to good faith of a nature Alhambra Cigar v. Mojica
calculated to discredit the goods, Unfair competition consists in passing off or
business or services of another attempting to pass off upon the public the
goods or business of one person as and for
168.4. The remedies provided by Sections the goods or business of another. It consists
156, 157 and 161 shall apply mutatis essentially in the conduct of a trade or
mutandis. business in such a manner that there is
either an express or implied representation
Section 168.1 speaks of a person who has to that effect.
earned goodwill with respect to his goods
and services and who is entitled to Coca-Cola Bottlers v. Gomez
protection under the Code, with or without a Jurisprudence defines unfair competition as
registered mark. the passing off or attempting to pass off
upon the public the goods or business of
Goodwill – property right that must be one person as the goods or business of
protected another with the end and probable effect of
Section 168.2 refers to the general definition deceiving the public.
of unfair competition.
True test of Unfair Competition: whether the
Form of unfair competition – passing off acts of defendant are such as are calculated
to deceive the ordinary buyer making his
Note: another form of unfair competition is
purchases under the ordinary conditions
false designation of origin
which prevail in the particular trade to which
Section 168.3, on the other hand, refers to the controversy relates.
the specific instances of unfair competition,
with Section 168.1 referring to the sale of Levi Strauss v. Tony Lim
goods given the appearance of the goods of Unfair competition consists in employing
another; deception or any other means contrary to
good faith by which any person shall pass
Section 168.2, to the inducement of belief off the goods manufactured by him or in
that his or her goods or services are that of which he deals, or his business, or services
another who has earned goodwill; while the for those of the one having established
disputed Section 168.3 being a “catch all” goodwill, or committing any acts calculated
clause whose coverage the parties now to produce such result.
dispute.
Elements of Unfair Competition
1. Create/capable of creating confusion as
Trademark Unfair to the enterprise, goods, industrial
Infringement Competition activities
Unauthorized use of Passing off of one’s 2. Formulate false allegations in the course
a trademark goods as those of of trade so as to discredit the enterprise,
another goods, industrial activities
Fraudulent Intent is Fraudulent intent is 3. Indications or allegations that in the
unnecessary. essential. course of trade….
Prior registration of Registration is NOT
the trademark is a necessary.
prerequisite to the
action.
Intellectual Property Law ►►| 38

Jurisprudential Elements of Unfair admitted to entry at any customhouse of


Competition the Philippines. In order to aid the officers
of the customs service in enforcing this
NBI-Microsoft Corporation v. Judy prohibition, any person who is entitled to
Hwang the benefits of this Act, may require that his
The elements of unfair competition name and residence, and the name of the
under Article 189(1) of the Revised Penal locality in which his goods are
Code are: manufactured, a copy of the certificate of
1. That the offender gives his goods the registration of his mark or trade name, to
general appearance of the goods of be recorded in books which shall be kept
another manufacturer or dealer; for this purpose in the Bureau of Customs,
2. That the general appearance is shown under such regulations as the Collector of
in the (1) goods themselves, or in the (2) Customs with the approval of the Secretary
wrapping of their packages, or in the (3) of Finance shall prescribe, and may furnish
device or words therein, or in (4) any to the said Bureau facsimiles of his name,
other feature of their appearance [;] the name of the locality in which his goods
3. That the offender offers to sell or sells are manufactured, or his registered mark or
those goods or gives other persons a trade name, and thereupon the Collector of
chance or opportunity to do the same Customs shall cause one (1) or more
with a like purpose[; and] copies of the same to be transmitted to
4. That there is actual intent to deceive the each collector or to other proper officer of
public or defraud a competitor. the Bureau of Customs.
Note: Art. 189 of RPC was already repealed
by the IP code
In and Out Burger v. Sehwani Once you are registered, you can give your
The essential elements of an action for certification your certificate of registration to
unfair competition are: the Bureau of Customs so they can monitor
1. Confusing similarity in the general goods going in and out of the country
appearance of the goods and bearing similar marks.
2. Intent to deceive the public and defraud
a competitor.
RA 9502, Universally Accessible Cheaper
and Quality Medicines Act of 2008
BUREAU OF CUSTOMS RECORDAL
SEC. 15. Section 159 of Republic Act No. 8293,
IP Code, Section 166 otherwise known as the Intellectual Property
Goods Bearing Infringing Marks or Trade Code of the Philippines, is hereby amended to
Names. — read as follows:
No article of imported merchandise which
SEC. 159. Limitations to Actions for Infringement. -
shall copy or simulate the name of any
Notwithstanding any other provision of this Act, the
domestic product, or manufacturer, or
dealer, or which shall copy or simulate a remedies given to the owner of a right infringed
mark registered in accordance with the under this Act shall be limited as follows:
provisions of this Act, or shall bear a 159.4 There shall be no infringement of trademarks
mark or trade name calculated to induce or tradenames of imported or sold drugs and
the public to believe that the article is medicines allowed under Section 72.1 of this Act, as
manufactured in the Philippines, or that it well as imported or sold off-patent drugs and
is manufactured in any foreign country or medicines: Provided, That said drugs and medicines
locality other than the country or locality bear the registered marks that have not
where it is in fact manufactured, shall be
Intellectual Property Law ►►| 39

been tampered, unlawfully modified, or infringed McDonald’s Corp. v. L.C. Big Mak
upon as defined under Section 155 of this Code. Big Mak Burger Inc. applied for the
registration of the mark “BIG MAK” this was
opposed by McDonalds as this was a
colorable imitation of its mark “Big Mac” over
FALSE DESIGNATION OF ORIGIN its double-decker hamburger sandwich. Big
Mak Inc. pointed out that another corp.(Isaiyas
IP Code, Section 169 Group) registered the mark “Big Mac” ahead
False Designations of Origin; False of McDonalds.In response McDonalds pointed
Description or Representation – out that Isaiyas Group’s registration was only
169.1. Any person who, on or in in the supplemental register; Also there was a
connection with any goods or services, deed of assignment made by Isaiyas Group in
or any container for goods, uses in favor of McDonalds. Trial court ruled that Big
commerce any word, term, name, symbol, Mak Inc is liable for infringement and unfair
or device, or any combination thereof, or competition. This was reversed by CA.
any false designation of origin, false or
misleading description of fact, or false or
misleading representation of fact, which:
The Court ruled that LC Big Mak is guilty of
(a) Is likely to cause confusion, or to infringement and unfair competition. LC Big
cause mistake, or to deceive as to the Mak was deemed guilty of infringement
affiliation, connection, or association because of likelihood of confusion as to
of such person with another person, goods and business and because it failed to
or as to the origin, sponsorship, or put notice that LC Big Mak is the
approval of his or her goods, manufacturer of Big Mak burgers.
services, or commercial activities by
another person; or
Distinguish Generic, Descriptive and
(b) In commercial advertising or Fanciful/ Arbitrary marks.
promotion, misrepresents the nature,
Generic marks are commonly used as the
characteristics, qualities, or
name or description of a kind of goods, such
geographic origin of his or her or
as “Lite” for beeror “Chocolate Fudge” for
another person's goods, services, or
chocolate soda drink.
commercial activities, shall be liable
to a civil action for damages and
Descriptive marks, on the other hand,
injunction provided in Sections 156
convey the characteristics, functions,
and 157 of this Act by any person
qualities or ingredients of a product to one
who believes that he or she is or is
who has never seen it or does not know it
likely to be damaged by such act.
exists, such as “Arthriticare” for arthritis
medication.
169.2. Any goods marked or labelled in
contravention of the provisions of this
On the contrary, “Big Mac” falls under the
Section shall not be imported into the
class of fanciful or arbitrary marks as it
Philippines or admitted entry at any
bears no logical relation to the actual
customhouse of the Philippines. The
characteristics of the product it represents.
owner, importer, or consignee of goods
refused entry at any customhouse under
Is there confusion of business in the
this section may have any recourse under
case?
the customs revenue laws or may have the
YES. Respondents assert that their “Big
remedy given by this Act in cases involving
Mak” hamburgers cater mainly to the low-
goods refused entry or seized.
income group while petitioners’ “Big Mac”
hamburgers cater to the middle and upper
income groups. Even if this is true, the
Intellectual Property Law ►►| 40

likelihood of confusion of business given a plausible reason for how they came
remains, since the low-income group might up with their mark.
be led to believe that the “Big Mak”
hamburgers are the low-end hamburgers Explain now Respondents came to
marketed by petitioners. After all, petitioners choose their mark. How did the Court
have the exclusive right to use the “Big react to this?
Mac” mark. On the other hand, respondents “MACJOY” is based on the name of its
would benefit by associating their low-end president’s niece, Scarlett Yu Carcell as an
hamburgers, through the use of the “Big expression of fondness and endearment.
Mak” mark, with petitioners’ high-end “Big
Mac” hamburgers, leading to likelihood of The court found the explanation to be
confusion in the identity of business. implausible. The respondent’s implausible
and insufficient explanation as to how and
Respondents have adopted in “Big Mak” not why out of the many choices of words it
only the dominant but also almost all the could have used for its trade name and/or
features of “Big Mac.” They both sound the trademark, it chose the word “MACJOY,”
same and that the letters are the same lead to the only logical conclusion deducible
except for the last (in tagalog “k” is used therefrom that the respondent would want to
instead of “c” as in Kalookan) ride high on the established reputation and
goodwill of the MCDONALD’s marks, which,
Explain how Respondents came to as applied to petitioner’s restaurant
choose their mark. business and food products, is undoubtedly
Defendants-appellants explained that the beyond question.
name “Mak” in their corporate name (BIG
MAK) was derived from both the first names of Prosource International, Inc. v. Horphag
the mother and father of defendant Francis Research
Dy, whose names are Maxima and Kimsoy. Horphag is a Swiss Corp. distributing food
supplement product in the Philippines with
Is it possible to have trademark the mark PYCNOGENOL. It discovered that
infringement without unfair competition? Prosource was also selling food supplement
Cite example. products with the mark PCO-GENOLS and
YES. There can be trademark infringement it demanded that the latter cease from
without unfair competition as when the distributing such products. Prosource the
infringer discloses on the labels containing products with PCO-GENOLS mark from the
the mark that he manufactures the goods, market and changed it into PCO-PLUS.
thus preventing the public from being Horphag then filed a case for infringement.
deceived that the goods originate from the The Court held that there was an
trademark owner. infringement because applying the
dominancy test there is likelihood of
McDonald’s Corp. v. Macjoy Fastfood confusion as the two marks sounds similar
Corp., and are both food supplement products.
MacJoy Fastfood is a domestic corp in Cebu
engaged in the fast food business and it What test was used in the case?
applied for registration of the mark “MACJOY Dominancy test. The court adhered to the
AND DEVICE” McDonalds opposed the findings of both the trial court and CA which
application saying that there is colorable both used the dominancy test. The products
imitation of their corporate logo in specifically are similar as to sounds; and both are food
the golden arches or the “M” sign. The Court supplements.
applied the dominancy test and held that the
marks were similar and MacJoy has not
Intellectual Property Law ►►| 41

Skechers USA v. Pacific Industrial exclusive right to use the mark to the
Trading prejudice of petitioner.
Skechers applied for search warrants to The GALLO trademark registration
search the warehouses of Pacific Industrial on certificates in the Philippines and in other
the ground of infringement; search warrants countries expressly state that they cover
were issued. On its defense, Pacific industrial wines only, without any evidence or
that there is no confusing similarity between indication that registrant Gallo Winery
the skechers rubber shoes and its own Strong expanded or intended to expand its
rubber shoes; hence it moved for the quashal business to cigarettes. Thus, by strict
of the search warrants. This was granted. The application of Section 20 of the Trademark
Court held that there was similarity with regard Law, Gallo Winery’s exclusive right to use
to the dominant feature of the skechers rubber the GALLO trademark should be limited to
shoes which is the stylized “S” and hence the wines, the only product indicated in its
quashal of the warrants was set aside. registration certificates.

REVIEW OF UNFAIR COMPETITION


What is the dominant feature of the mark?
CASES UNDER TRADEMARK LAW
Is it registrable?
The dominant feature of the mark of McDonalds Corp. v. Macjoy Fastfood
Skechers is the stylized “S” within an oval Corp.
design. YES in this case the Court
There was a recognition of House Marks
mentioned that indeed Skechers has a
(family of “Mc” marks of McDonalds were
registered mark for the stylized “S”.
impliedly recognized in this case).
Mighty Corporation v. E.J. Gallo Winery
Superior Enterprises v. Kunnan
Gallo Winery is a foreign corp and it has
Enterprises
registered trademark GALLO of its wine
Another type of unfair competition other
products here in the Philippines. Mighty Corp
than passing off and false designation of
and La Compana are domestic corporations
origin was shown in this case: discredit of
engaged in the manufacture and sale of
the goods, business and services of another
tobacco products and they later acquired the
(IP Code, Sec. 168c)
right to the registered trademark GALLO
earlier registered by Tobacco industries. Gallo
Willaware Products v. Jesichris
winery filed a complaint for infringement and
Manufacturing
unfair competition against Mighty Corp and La
UC under the IP Code and Art. 28 Civil
Compana alleging that the two wanted to ride
Code (Unfair Trade Practices).
on the goodwill of its wine products. The Court
held that there was no infringement because Art. 28 of the Civil Code could serve as a
wines and cigarettes are not identical, similar basis for a claim if unfair competition under
or related goods; that Gallo Winery’s
the IP Code is not an option.
registration covered wines only and that
GALLO is not an internationally well-known Chester Uyco v. Lo
mark under the Paris Convention. There was UC and False designation of origin
no unfair competition because Mighty Corp. (products indicated that they were made in
never intended to pass off their cigarette Portugal when in fact they were
products as that of Gallo winery. manufactured in the PH)

Explain the reasoning of the Court in


concluding that Respondents never had
Intellectual Property Law ►►| 42

MADRID PROTOCOL (Protocol


INTERNATIONAL APPLICATION AND
Relating to the Madrid Agreement INTERNATIONAL REGISTRATION
Concerning the International Q: What is the difference between an
Registration of Marks) international application and
international registration?
Only an executive agreement, hence no A: International application refers to the
need for 2/3 concurrence vote of the Senate stage where the applicant files in its office of
(IPAP v. Ochoa) origin the basic application which will be
forwarded to the IPO for formal
Effective date in PH: July 25, 2012 examination; or the stage before the
trademark is registered in the WIPO
Philippines’ reservations/declarations when international register of marks
it acceded to the Protocol:
1. Provisional Refusal: 18months + International registration refers to the
stage where after the formal examination,
2. No subsequent designations the trademark is registered in the WIPO
3. IPOPHIL collects individual fee international register; then the application
would be forwarded to the DCPs
Q: Is there a conflict between the Madrid
(Designated Contracting Parties)
Protocol and the IP Code? (On the ground
that Madrid Protocol does away with the
Note: Certificate of Registrations (CORs)
resident agent requirement under the IP
are not issued by IPOPHIL for Madrid
Code) (IPAP v. Ochoa)
Applications but only Statement of Grant of
A: None. The Madrid Application would still
Protection
be subject to the same substantive
requirements under our laws.
ROLE OF ATTORNEYS (Sec.125)
A resident agent/local representative would
IP Code, Sec. 125
still be required when:
Representation; Address for Service.
1. the IPOPHIL refuses registration of the
If the applicant is not domiciled or has no
mark;
real and effective commercial
2. when the submission of the DAU is
establishment in the Philippines, he shall
required and
designate by a written document filed in
3. also upon submission of the license the Office, the name and address of a
contract Philippine resident who may be served
notices or process in proceedings
COMMON PROVISIONS FOR BOTH
affecting the mark. Such notices or
MADRID AND NON-MADRID services may be served upon the person
APPLICATIONS so designated by leaving a copy thereof
1. Substantive examination in accordance at the address specified in the last
with PH Laws designation filed. If the person so
2. If Registration Report is issued, applicant
designated cannot be found at the
must be represented by a local agent address given in the last designation,
3. When required to submit DAU, applicant such notice or process may be served
must be represented by local agent upon the Director.
4. Submission and recordal of assignment:
license contracts before the IPO
ROLE OF IPO
1. As Office of Origin (Inbound
HOW MADRID PROTOCOL WORKS
applications)- help clients in reviewing
(See chart at the last page)
the documents, review and filing of the
Intellectual Property Law ►►| 43

Madrid Application through IPO; Class Items


General advices Class 5 Pharmaceutical, veterinary and
2. As Designated Contracting Party (DCP) sanitary preparations; dietetic
(Outbound applications) – reviewing substances adapted for medical use,
Form MM2 which must contain: food for babies; plasters, materials
- Name and contact details of for dressings; material for stopping
applicant teeth, dental wax; disinfectants;
- Reproduction of trademark preparations for destroying vermin;
- Specification of goods or service fungicides, herbicides
Class 6 Common metals and their alloys;
- Designated contracting parties metal building materials;
transportable buildings of metal;
materials of metal for railway tracks;
NICE CLASSIFICATION non-electric cables and wires of
common metal; ironmongery, small
The following are the general classifications items of metal hardware; pipes and
of goods (Classes 1-34) and services tubes of metal; safes; goods of
(Classes 35-45). common metal not included in other
Class Items classes;ores.
Class 1 Chemicals used in industry, science Class 7 Machines and machine tools; motors
and photography, as well as in and engines (except for land
agriculture, horticulture and forestry; vehicles); machine coupling and
unprocessed artificial resins, transmission components (except for
unprocessed plastics; manures fire land vehicles); agricultural
extinguishing compositions; implements other than hand-
tempering and soldering operated; incubators for eggs.
preparations; chemical substances Class 8 Hand tools and implements (hand-
for preserving foodstuffs; tanning operated); cutlery; side arms; razors.
substances; adhesives used in Class 9 Scientific, nautical, surveying,
industry electric, photographic,
cinematographic, optical, weighing,
Class 2 Paints, varnishes, lacquers; measuring, signaling, checking
preservatives against rust and (supervision), life-saving and
against deterioration of wood; teaching apparatus and instruments;
colorants; mordants; raw natural apparatus for recording, transmission
resins; metals in foil and powder form or reproduction of sound or images;
for painters, decorators, printers and magnetic data carriers, recording
artists. discs; automatic vending machines
and mechanisms for coin-operated
Class 3 Bleaching preparations and other apparatus; cash registers, calculating
substances for laundry use; cleaning, machines, data processing
polishing, scouring and abrasive equipment and computer; fire-
preparations; soaps; perfumery, extinguishing apparatus
essential oils, cosmetics, hair lotions; Class 10 Surgical, medical, dental and
dentifrices. veterinary apparatus and
Class 4 Industrial oils and greases; instruments, artificial limbs, eyes and
lubricants; dust absorbing, wetting teeth; orthopedic articles; suture
and binding compositions; fuels materials.
(including motor spirit) and Class 11 Apparatus for lighting, heating, steam
illuminants; candles, wicks. generating, cooking, refrigerating,
Intellectual Property Law ►►| 44

Class Items Class Items


drying, ventilating, water supply and horn, bone, ivory, whalebone, shell,
sanitary purposes amber, mother-of-pearl,
Class 12 Vehicles; apparatus for locomotion meerschaum and substitutes for all
by land, air or water. these materials, or of plastics.
Class 13 Firearms; ammunition and Class 21 Household or kitchen utensils and
projectiles; explosives; fireworks. containers (not of precious metal or
Class 14 Precious metals and their alloys and coated therewith); combs and
goods in precious metals or coated sponges; brushes (except paint
therewith, not included in other brushes); brush-making materials;
classes; jewellery, precious stones; articles for cleaning purposes;
horological and chronometric steelwool; unworked or semi-worked
instruments glass (except glass use in building);
Class 15 Musical instruments glassware, porcelain and
Class 16 Paper, cardboard and goods made earthenware not included in other
from these materials, not included in classes.
other classes; printed matter; Class 22 Ropes, string, nets, tents, awnings,
bookbinding material; photographs; tarpaulins, sails, sacks and bags (not
stationery; adhesives for stationery included in other classes); padding
or household purposes; artist's and stuffing materials (except of
materials; paint brushes; typewriters rubber or plastics); raw fibrous textile
and office requisites (except materials.
furniture); instructional and teaching Class 23 Yarns and threads, for textile use.
material (except apparatus); plastic Class 24 Textiles and textile goods, not
materials for packaging (not included included in other classes; bed and
in other classes); playing cards; table covers.
printers' type printing blocks. Class 25 Clothing, footwear, headgear.
Class 17 Rubber, gutta-percha, gum, Class 26 Lace and embroidery, ribbons and
asbestos, mica and goods made braid; buttons, hooks and eyes, pins
from these materials and not and needles; artificial flower
included in other classes; plastics in Class 27 Carpets, rugs, mats and matting,
extruded form for use in linoleum and other materials for
manufacture; packing, stopping and covering existing floors; wall
insulating materials; flexible pipes, hangings (non-textile).
not of metal. Class 28 Games and playthings; gymnastic
Class 18 Leather and imitations of leather, and and sporting articles not included in
goods made of these materials and other classes; decorations for
not included in other classes; animal Christmas trees
skins, hides; trunks and traveling Class 29 Meat, fish, poultry and game; meat
bags; umbrellas, parasols and extracts; preserved, dried and
walking sticks; whips, harness and cooked fruits and vegetables; jellies,
saddlery. jams, fruit sauces; eggs, milk and
Class 19 Building materials (non-metallic); milk products; edible oils and fats.
non-metallic rigid pipes for building; Class 30 Coffee, tea, cocoa, sugar, rice
asphalt, pitch and bitumen; non- tapioca, sago, artificial coffee; flour
metallic transportable buildings; and preparations made from cereals,
monuments, not of metal. bread, pastry and confectionery,
Class 20 Furniture, mirrors, picture frames; ices; honey, treacle; yeast, baking-
goods (not included in other classes) powder; salt, mustard; vinegar,
of wood, cork, reed, cane, wicker, sauces (condiments); spices; ice.
Intellectual Property Law ►►| 45

Class Items the Madrid Protocol where the


Class 31 Agricultural, horticultural and forestry Philippines is a DCP. Can the trademark
products and grains not included in owner exclude third parties from using
other classes; live animals; fresh the mark registered under the
fruits and vegetables; seeds, natural International Register?
plants and flowers; foodstuffs for A: NO. Registration of the mark in the
animals; malt. International Register does not ipso facto
Class 32 Beers; mineral and aerated water give the trademark owner the right to
and other non-alcoholic drinks; fruit exclude parties from using the mark. The
drinks and fruit juices; syrups and trademark owner must apply for the
other preparations for making extension of the territorial protection of the
beverages. mark to the other contracting parties of the
Class 33 Alcoholic beverages (except beers). Madrid Protocol and the Paris Convention
Class 34 Tobacco; smokers' articles; matches for Protection of Industrial Property.
Class 35 Advertising; business management;
business administration; office
functions. THE LAW ON PATENTS
Class 36 Insurance; financial affairs; monetary
affairs; real estate affairs.
Class 37 Building construction; repair; INTRODUCTION
installation services.
Class 38 Telecommunications. Law on Patents is basically the interplay
Class 39 Transport; packaging and storage of between two concepts (confidential
goods; travel arrangement. information and Publication/disclosure).
Class 40 Treatment of materials.
Class 41 Education; providing of training; CONFIDENTIAL INFORMATION
entertainment; sporting and cultural
activities.
(e.g. Trade Secret)
Class 42 Scientific and technological services
1. Requisites for Liability
and research and design relating
thereto; industrial analysis and a. Subject matter that is capable of
research services; design and protection
development of computer hardware b. Confidential Obligation
and software; legal services. c. Unauthorized use
Class 43 Services for providing food and drink; 2. Remedies
temporary accommodation a. Injunction
Class 44 Medical services, veterinary b. Damages and other monetary reliefs
services; hygienic and beauty care
for human beings or animals; Examples of Patents: design details,
agriculture; horticulture and forestry manufacturing processes and
services. Product/chemical composition
Class 45 Personal and social services
rendered by others to meet the needs WAYS OF PROTECTING CONFIDENTIAL
of individuals; security services for INFORMATION (also types of confidential
the protection of property and obligation)
individuals. 1. Legal – Attorney-client privilege
Note: are paralegals covered?
2. Contractual
Q: A mark was just issued an a. Non-disclosure agreements –
International Registration number under especially during negotiations with
Intellectual Property Law ►►| 46

other companies for manufacturing 3. The 20th century show process if


or mass production of the product internationalization in terms of
b. Restrictive covenants – applies to substantive law
employees (e.g. prohibited from - Standards on what is new or novel
being employed with another
pharmaceutical company for a a. Greeks
limited time and place) b. Venice – first formal law on patents
c. England – where industrial revolution
started
CHARACTERISTICS OF A PATENT
Pearl & Dean v. Shoemart
To be able to effectively preclude others JUSTIFICATION OF THE PATENT
from profiting from the invention, a patent is SYSTEM
a primordial requirement. No patent, no
protection. THREEFOLD PURPOSE
1. Foster and reward invention
Ideas which are disclosed to the public
without the protection of a valid patent are 2. Promote disclosures of invention to
subject to appropriation. stimulate further inventions and to
permit the public to practice the
Ching v. Salinas invention
Works of applied art 3. Stringent requirements for patent
General Rule: works of applied art are protection seek to ensure that ideas
copyrightable (Copyright Law) in the public domain remain there for
the free use of the public
Exception: If it has aesthetic feature and
utilitarian function and you cannot separate Ultimate goal of a patent system: to bring
the two then patentable (Patent law) new designs and technology into the public
domain through disclosure
Implication: If it can be separated then, the
aesthetic feature – copyrightable while the INTERNATIONAL AGREEMENTS
utilitarian function – patentable
MAIN FEATURES OF THE PARIS
CONVENTION
HISTORY OF THE PATENT 1. Principle of National Treatment
SYSTEM Each signatory state will afford the same
rights to foreigners of other signatory
states to its own nationals
KEY POINTS ON EARLY HISTORY
2. Applicant enjoys a 12-month right of
1. A form of social contract with obligations priority from the date of first filing of a
on both sides. patent in a signatory country. It also
State: patent protection (give monopoly means novelty of all such applications
rights) will be tested by reference to the period
Inventor: disclose invention before the first filing.
2. Monopoly is viewed with suspicion 3. National patent remain independent of
- No standards before; patronage and each other. (Art. 4bis)
favoritism; monopoly was given only
with new inventions
Intellectual Property Law ►►| 47

4. The focus on the inventor who has the available for any inventions, whether
right to be named as such in the patent. products or processes, in all fields of
(Art. 4ter) technology, provided that they are
Patent ownership: new, involve an inventive step and
a. Employer-employee – if part of the are capable of industrial application.
employee’s regular work – employer Subject to paragraph 4 of Article 65,
(through a deed of assignment paragraph 8 of Article 70 and
affixed in the patent application) paragraph 3 of this Article, patents
b. Commissioned work - commissioner shall be available and patent rights
enjoyable without discrimination as
to the place of invention, the field of
THE PATENT COOPERATION TREATY technology and whether products are
AS AMENDED imported or locally produced.
1. Not a granting procedure — merely
provides a conduit whereby a single FOUR BASIC REQUIREMENTS FOR
application can be processed and PATENTABILITY
searched by a qualified international 1. There must be an invention that can be
body before being sent to national or considered as a technical solution of a
regional intellectual property offices for problem in any field of human activity;
full consideration
2. The invention must be new;
2. Anyone who is a national or resident of
a contracting state can file a single 3. The invention must involve an inventive
international patent application step;
Note: This single application is then 4. The invention must be industrially
subject to an ‘international search’ from applicable;
which an international research report is
prepared.
This international search body PATENTABLE AND NON-
determines if there is inventive step; can PATENTABLE INVENTIONS
help you decide whether to pursue your
international application
PART A:
Difference with the Madrid Protocol: you Define
deal directly with the national Intellectual
Property Offices 1. Invention
IP Code, Section 21
Patentable Inventions
TRIPS (1994) Any technical solution of a problem in
1. It is administered by the WTO and any field of human activity which is new,
parties to the Agreement are subject to involves an inventive step and is
WTO’s dispute settlement system industrially applicable shall be
2. It links obligations to protect intellectual patentable. It may be, or may relate to, a
property with the attractiveness of trade product, or process, or an improvement
privileges available under GATT of any of the following.
3. First international measure to harmonize - Technical character
substantive patent provisions: - Solution
2. Product – any physical thing resulting
Article 27(1) TRIPS from a man’s skill
Patentable Subject Matter 3. Process – mode of treatment of certain
1. Subject to the provisions of materials to produce a certain effect
paragraphs 2 and 3, patents shall be
Intellectual Property Law ►►| 48

Only limitation: Laws of nature 5. Aesthetic creations


6. Contrary to public order or morality
Not patentable:
A new mineral discovered in the BAR Q: Supposing Albert Einstein were
earth A new plant found in the wild alive today and he filed with the
Einstein’s E=mc2 Intellectual Property Office an
Newton could not have patented the law application for patent of his theory of
on gravity. relativity expressed in the formula
E=mc2.The IPO disapproved Einstein
application on the ground that his theory
of relativity is not patentable
PART B:
Is the IPO action correct?
Classes of Patentable Inventions A: Yes, the IPO's action is correct that the
theory of relativity is not patentable. Under
1. A product, such as machine, a device, section 22.1 of the IPC. "discoveries,
an article of manufacture, a composition scientific theories and mathematical
of matter, a microorganism methods" are not patentable.
2. A process, such as method of use, a
method of manufacturing, a non- BAR Q: Dr. Nobel discovered a new
biological process, a microbiological method of treating Alzheimer’s involving
process; a special method of diagnosing the
3. Computer-related inventions; and disease, treating it with a new medicine
Note: Computer programs per se not that has been discovered after long
patentable; if part of a process or when experimentation and field testing, and
related to a machine. Equipment of novel mental isometric exercises. He
procedure or a component of a whole comes to you for advice on how he can
machine it is patentable. have his discoveries protected. Can he
4. An improvement of any of the foregoing. legally protect his new method of
diagnosis, the new medicine, and the
NON-PATENTABLE INVENTIONS new method of treatment? If no, why? If
- Abstract (22.1 and 22.2) yes, how? (4%)
- Other non-technical things (22.5) A: Dr. Nobel can be protected by a patent
for the new medicine as it falls within the
1. Discoveries, scientific theories and scope of Sec. 21 of the Intellectual Property
mathematical methods Code (Rep. Act No. 8293, as amended). But
Note: Including known derivatives of a no protection can be legally extended to him
known substance unless they differ for the method of diagnosis and method of
significantly in properties with regard to treatment which are expressly non-
efficacy patentable (IP Code, Sec.22).
2. Schemes, rules and methods, for
performing mental acts, playing games
or doing business, and programs for Discoveries, scientific theories
computers Non-Patentable: new property of a known
3. Methods for treatment of the human or material or article, that is a mere discovery
animal body
4. Plant varieties or animal breeds, Patentable: person puts that property to
essentially biological processes for plant practical use, an invention has been made,
or animal production which may be patentable
Note: Does not apply to micro-
organisms and non-biological or
microbiological processes
Intellectual Property Law ►►| 49

Such discoveries are “manifestations of Patentable: used by a locksmith in case of


nature, free, to all men and reserved emergency
exclusively to none.”
A fair test to apply is to consider whether it is
Phenomena of nature, although just probable that the public in general would
discovered, mental processes, and abstract regard the invention as so abhorrent that the
intellectual concepts are not patentable, as grant of patent rights would be inconceivable
they are the basic tools of scientific and
technological work.
Plant varieties or animal breeds,
essentially biological processes for plant
Mathematical methods Non-Patentable: a and animal production
shortcut method of division would not be Not-patentable: no significant intervention of
patentable man
Patentable: if such intervention plays a
Patentable: a calculating machine construed
significance part in determining or
to operate the above described shortcut
controlling the result is desired to achieve
method of division
Patentable Microbiological Process
Schemes, rules and methods for mental
- industrial processes under micro-
acts
organisms
Non-Patentable: a scheme for learning a
- processes for producing new micro-
language, a method of solving cross-word organisms
puzzles, a game (as an abstract entity - product of a microbiological process may
defined by its rules) or a scheme for be also patentable per se (product claim)
organizing a commercial operation would - micro-organism can also be protected
not be patentable per se (IP Code, Sec. 22.4[2])
Patentable: novel apparatus for playing a
Methods for treatment of the human or
game or carrying out a scheme might be
animal body by surgery or therapy Not-
patentable
patentable: methods of treatment by surgery
or therapy and diagnostic method
Programs for computers Non-Patentable:
a computer program claimed by itself or as
Patentable: other methods of treatment of
a record on a carrier or when the computer
live human beings or animals or other
program is loaded into a known computer
methods of measuring or recording
characteristics of the human or animal body
Patentable: a computer program-controlled
are patentable provided that such methods
process should be regarded as patentable
are of a technical and are not essentially
subject matter (practical use)
biological in character
Contrary to public order or morality
Q: Is this excluded: a treatment of or
Non-Patentable: letter bomb
diagnostic method practiced on dead
Patentable: guns and ammunitions human or animal body?
A: NO. Only methods of surgery, treatment
Q: What if the invention is a process of and diagnosis on live human or animal are
breaking open locked safes? excluded from patent protection under Sec.
A: Not-patentable: to facilitate a burglar’s 22 of the IP Code.
robbery activity
Intellectual Property Law ►►| 50

PATENTABILITY both applications are not one and the


same.
PART 1: IP Code, Section 25.
Novelty Non-Prejudicial Disclosure. —
25.1. The disclosure of information
IP Code, Section 23, contained in the application during the
Novelty. — twelve (12) months preceding the filing
An invention shall not be considered new date or the priority date of the application
if it forms part of a prior art. shall not prejudice the applicant on the
ground of lack of novelty if such
Q: What constitutes prior art? disclosure was made by:
1. Available to the public anywhere in the a. The inventor;
world before the filing date and the b. A patent office and the information
priority date was contained (a) in another
2. Contents of an application with earlier application filed by the inventor and
filing or priority date than the subject should not have been disclosed by
application the office, or (b) in an application
filed without the knowledge or
Revised Rules for Patent Utility, Rule 204.1 consent of the inventor by a third
Equivalents – a strict identity test is party which obtained the
required to be applied in assessing information directly or indirectly
novelty. A single prior art reference must from the inventor; or
disclose each and every element of a c. A third party which obtained the
claimed invention in order to destroy information directly or indirectly
novelty. Equivalents are considered only from the inventor.
in assessing inventive step.
25.2. For the purposes of Subsection
IP Code, Section 24 25.1, "inventor" also means any person
Prior Art. — who, at the filing date of application, had
Prior art shall consist of: the right to the patent.
24.1. Everything which has been made
available to the public anywhere in the Manzano v. Court of Appeals
world, before the filing date or the priority An invention must possess the essential
date of the application claiming the elements of novelty, originality and
invention; and precedence, and for the patentee to be
entitled to the protection the invention must
24.2. The whole contents of an application be new to the world.
for a patent, utility model, or industrial
design registration, published in The element of novelty is an essential
accordance with this Act, filed or effective requisite of the patentability of an invention or
in the Philippines, with a filing or priority discovery. If a device or process has been
date that is earlier than the filing or priority known or used by others prior to its invention
date of the application: Provided, That the or discovery by the applicant, an application
application which has validly claimed the for a patent therefor should be denied; and if
filing date of an earlier application under the application has been granted, the court, in
Section 31 of this Act, shall be prior art with a judicial proceeding in which the validity of
effect as of the filing date of such earlier the patent is drawn in ques/on, will hold it void
application: Provided further, That the and ineffective. It has been repeatedly held
applicant or the inventor identified in that an invention must possess the
Intellectual Property Law ►►| 51

essential elements of novelty, originality and


precedence Aguas v. De Leon
Tile Making — Where improvement in /le
PART 2: making is inventive and different from old
Inventive Step process of tile making, improvement is
patent able.
IP Code, Section 26
Inventive Step – Moreover, it appears that appellee has been
26.1. An invention involves an inventive deriving considerable profit from his
step if, having regard to prior art, it is not manufacture and sale of such tiles. This
obvious to a person skilled in the art at commercial success is evidence of
the time of the filing date or priority date patentability
of the application claiming the invention.

26.2. In case of drugs and medicines, there Graham v. John Deere Co.
is no inventive step if the invention results Ruling:
from mere discovery of a new form or new 1. The flexing advantages flowing from the
property of a known substance which does ‘798 arrangement are not, in fact, a
not result in the enhancement of the known significant feature of the patent.
efficacy of that substance, or the mere 2. All of the elements of the ‘798 patent are
discovery of any new property or new use present in the Glencoe (clamp device).
for a known substance, or the mere use of a
known process unless such known process Graham Factors – used in determining non-
results in a new product that employs at obviousness:
least one new reactant. 1. Scope and content of prior art
2. Differences between the claimed
invention and the prior art
Vargas v. FM Yaptico
3. Level of ordinary skill in the prior art
The burden of proof to substantiate a charge
of infringement is with the plaintiff. Where,
Secondary considerations:
however, the plaintiff introduces the patent in
1. Commercial success
evidence, if it is in due form, it affords a prima
2. Long felt but unsolved needs
facie presumption of its correctness and
3. Failure of others
validity. The decision of the Commissioner of
Patents in granting the patent is always
presumed to be correct. The burden then
shifts to the defendant to overcome by PART 3:
competent evidence this legal presumption. Industrial Applicability

As said by the United States Supreme IP Code, Section 27


Court, "it has been repeatedly held by this An invention that can be produced and
court that a single instance of public use of used in any industry shall be industrially
the invention by a patentee for more than applicable. (n)
two years before the date of his application
for his patent will be fatal to the validity of
Additional cases on Patents:
the patent when issued."
Law of Nature/Biological Activity
We are certain that he has at least Diamond v. Charkabarty
demonstrated the public use of the Vargas
Whether the micro-organism in this case is
plow over two years prior to the application
an article of manufacture. YES.
for a patent.
Intellectual Property Law ►►| 52

The two patents at issue concern the use of


A method for developing a bacterium that thiopurine used to treat autoimmune
could break down multiple components of diseases.
crude oil. His claim ranged from the process
of developing the bacterium to the bacterium When the thiopurine drugs are ingested, the
itself and an inoculum in which the bacterium body metabolizes them, producing
was stored with a carrier material. metabolites in the bloodstream.

Claims: Because patients metabolize these drugs


1. Process of producing a bacterial differently, doctors found it difficult to
organism determine whether a particular patient’s
2. Method of using a bacterial organism dose is too high, risking harmful side
3. The bacterial organism itself effects, or too low, and so likely ineffective.

Why does the examiner allow 1 and 2 but Ruling:


not 3? The concentrations of metabolites in the blood
Because the bacterium itself appears and the likelihood that a thiopurine drugs
natural and part of the laws of nature hence dosage will likely cause harm or prove
not patentable. Patents cannot protect laws ineffective are laws of nature and hence not
of nature or physical phenomena. themselves patentable absent any added
feature showing that the processes are
In this case, the patent over the bacterium genuine application of those laws rather than
itself was held valid because the law on efforts to monopolize the relations.
patent intended to separate products of Note: otherwise, it would constitute an
nature from products of human ingenuity. inhibition from using those laws of nature in
The bacterium was a product of making further discoveries.
Chakrabarty’s own ingenuity because it did
not occur in nature.
Abstract Ideas and Theories
As a rule of statutory construction, is a Diamond v. Diehr
bacterial organism a “composition of Is the process of using mathematical
nature” or “manufacture”? formula in this case a patentable subject
Naturally a composition of matter since it is matter? YES.
a natural living organism. Involves a computer which repeatedly
recalculates the cure time by the use of
Q: What is the rule of Chakrabarty? Arrhenius equation.
(What living things are patentable) If the computer use is incorporated in the
A: A live, human-made micro-organism, process then the entire process is not made
which constitutes a non-natural composition entirely unpatentable.
of matter or manufacture.
Summary:
1. Mathematical formulas cannot be
Treatment of Diseases patented in the abstract
Mayo Collaborative Services v. 2. When the claim containing a
Promotheus Laboratories Inc. mathematical formula implements the
Whether in adding the words “apply it” to a said formula as part of an entire process
known law of nature can make an invention then the process becomes patentable.
patentable. NO.
Is computer software patentable?
- Per se, subject to copyright
- But can be if part of a process
Intellectual Property Law ►►| 53

Specification (Sec.32 to 39)


Method of Doing Business: IP Code, Section 32
Bilski v. Kappos Filing The Application –
Whether the concept of hedging risk and the 32.1. The patent application shall be in
application of that concept to energy Filipino or English and shall contain the
markets are patentable processes. NO. following:
(a) A request for the grant of a patent;
The claimed invention explains how (b) A description of the invention;
commodities buyers and sellers in the (c) Drawings necessary for the
energy market can protect, or hedge, understanding of the invention;
against the risk of price changes. (d) One or more claims; and
(e) An abstract.
Federal court applied the machine or
transformation test. 32.2. No patent may be granted unless the
application identifies the inventor. If the
Machine or transformation test – a claimed applicant is not the inventor, the Office may
process is patent-eligible if: require him to submit said authority.
1. It is tied to a particular machine or
apparatus IP Code, Section 33.
2. It transforms a particular article into a Appointment of Agent or Representative
different state or thing An applicant who is not a resident of the
Note: not the sole test for patent eligibility Philippines must appoint and maintain a
resident agent or representative in the
Business methods are not categorically Philippines upon whom notice or process
excluded from patent protection. for judicial or administrative procedure
However the claims in this case are not relating to the application for patent or
patent-eligible because they represent an the patent may be served.
abstract idea.
IP Code Section 34.
The Request
The request shall contain a petition for the
APPLICATION PROCESS grant of the patent, the name and other data
of the applicant, the inventor and the agent
PATENT APPLICATION FLOW and the title of the invention.
CHART (See chart at the last page)
IP Code Section 35.
PROCEDURE FOR GRANT OF Disclosure and Description of the
PATENT Applicant Invention
IP Code, Section 28 35.1. Disclosure. –
Right to a Patent. - The application shall disclose the
The right to a patent belongs to the invention in a manner sufficiently clear
inventor, his heirs, or assigns. When two and complete for it to be carried out by a
(2) or more persons have jointly made an person skilled in the art.
invention, the right to a patent shall
belong to them jointly. Where the application concerns a
‘ microbiological process or the product
thereof and involves the use of a micro-
organism which cannot be sufficiently
disclosed in the application in such a way
as to enable the invention to be carried out
by a person skilled in the art, and such
Intellectual Property Law ►►| 54

material is not available to the public, the later application is filed within four (4)
application shall be supplemented by a months after the requirement to divide
deposit of such material with an becomes final, or within such additional
international depository institution. time, not exceeding four (4) months, as
may be granted: Provided further, That
35.2. Description. – each divisional application shall not go
The Regulations shall prescribe the beyond the disclosure in the initial
contents of the description and the order application.
of presentation. 8.3. The fact that a patent has been
granted on an application that did not
IP Code, Section 36. comply with the requirement of unity of
The Claims – invention shall not be a ground to cancel
36.1. The application shall contain one (1) the patent.
or more claims which shall define the
matter for which protection is sought. Each IP Code Section 39.
claim shall be clear and concise, and shall Information Concerning Corresponding
be supported by the description. Foreign Application for Patents –
The applicant shall, at the request of the
36.2. The Regulations shall prescribe the Director, furnish him with the date and
manner of the presentation of claims. number of any application for a patent filed
by him abroad, hereafter referred to as the
IP Code, Section 37. "foreign application," relating to the same
The Abstract – or essentially the same invention as that
The abstract shall consist of a concise claimed in the application filed with the
summary of the disclosure of the Office and other documents relating to the
invention as contained in the description, foreign application.
claims and drawings in preferably not
more than one hundred fifty (150) words.
It must be drafted in a way which allows
the clear understanding of the technical Priority Date
problem, the gist of the solution of that IP Code, Section 31
problem through the invention, and the An application for patent filed by any
principal use or uses of the invention. person who has previously applied for the
The abstract shall merely serve for same invention in another country which by
technical information. treaty, convention, or law affords similar
privileges to Filipino citizens, shall be
IP Code, Section 38 considered as filed as of the date of filing
38.1. The application shall relate to one the foreign application: Provided, That: (a)
invention only or to a group of inventions the local application expressly claims
forming a single general inventive priority; (b) it is filed within twelve
concept. (12) months from the date the earliest
foreign application was filed; and (c) a
38.2. If several independent inventions certified copy of the foreign application
which do not form a single general together with an English translation is
inventive concept are claimed in one filed within six (6) months from the date
application, the Director may require that of filing in the Philippines. (Sec. 15, R.A.
the application be restricted to a single No. 165a)
invention. A later application filed for an
invention divided out shall be considered
as having been filed on the same day as
the first application: Provided, That the
Intellectual Property Law ►►| 55

Filing Date eighteen (18) months from the filing date


It is accorded only when all the or priority date.
requirements under Sec.40 are present: 44.2. After publication of a patent
1. An express or implicit indication that a application, any interested party may
Philippine patent is sought inspect the application documents filed
2. Information identifying the applicant with the Office.
3. Description of the invention and one or 44.3. The Director General, subject to the
more claims in Filipino or English approval of the Secretary of Trade and
Industry, may prohibit or restrict the
Other possible requirements: publication of an application, if in his
1. Contents for the request for Philippine opinion, to do so would be prejudicial to
patent the national security and interests of the
2. Priority documents if with claim for Republic of the
convention priority Philippines. (n)
3. Proof of authority, if applicant is not the
inventor Confidentiality Before
4. Deed of assignment Publication IP Code, Section 45.
5. Payment of all fees A patent application, which has not yet
6. Signatures of the applicants been published, and all related
7. Identification of the inventor documents, shall not be made available
8. Formal drawings for inspection without the consent of the
applicant.
Formality Examination
After the patent application has been Rights Conferred After
accorded a filing date and the required fees Publication IP Code, Section 46
have been paid on time, the applicant shall Rights Conferred by a Patent Application
comply with the formal requirements under After Publication –
Sec.32 of the law and the implementing The applicant shall have all the rights of a
rules and regulations under the said section. patentee under Section 76 against any
person who, without his authorization,
exercised any of the rights conferred
under
Classification and Search Section 71 of this Act in relation to the
An application that has complied with the invention claimed in the published patent
formal requirements shall be classified and application, as if a patent had been
a search conducted to determine prior art. granted for that invention: Provided, That
- Examiner shall determine novelty the said person had:
- IPO uses International Patent
Classification 46.1. Actual knowledge that the invention
that he was using was the subject matter
Publication (before grant) of a published application; or
It should be made within 18 months from 46.2. Received written notice that the
filing or priority date. invention that he was using was the
subject matter of a published application
IP Code, Section 44 being identified in the said notice by its
Publication of Patent Application serial number:
44.1. The patent application shall be Provided, That the action may not be filed
published in the IPO Gazette together with until after the grant of a patent on the
a search document established by or on published application and within four (4)
behalf of the Office citing any documents years from the commission of the acts
that reflect prior art, after the expiration of complained of. (n)
Intellectual Property Law ►►| 56

Note: Section 76 - Granted that all the requirements of the


law and the implementing rules
Observation By Third Parties including payments of fees are satisfied
IP Code, Section 47 - Patent shall take effect on the date of
Following the publication of the patent the publication of the grant of patent in
application, any person may present the IPO gazette
observations in writing concerning the
patentability of the invention. Such Refusal of the Application
observations shall be communicated to - Possible that application will be refused
the applicant who may comment on them. because the grant is not matter of right
The Office shall acknowledge and put - Remedy is to appeal the decision of the
such observations and comment in the examiner to the Director of Bureau of
file of the application to which it relates. Patents

Substantive Examination
Duty of applicant to file a written request for Publication After Grant
substantive examination IP Code, Section 52
Publication Upon Grant of Patent
IP Code, Section 48 52.1. The grant of the patent together with
Request for Substantive Examination other related information shall be
48.1. The application shall be deemed published in the IPO Gazette within the
withdrawn unless within six (6) months time prescribed by the Regulations.
from the date of publication under 52.2. Any interested party may inspect
Section 41, a written request to determine the complete description, claims, and
whether a patent application meets the drawings of the patent on file with the
requirements of Sections 21 to 27 and Office.
Sections 32 to 39 and the fees have been
paid on time. Issuance of Certificate of Letters Patent
IP Code, Section 53
48.2. Withdrawal of the request for The patent shall be issued in the name of
examination shall be irrevocable and the Republic of the Philippines under the
shall not authorize the refund of any fee. seal of the Office and shall be signed by
Law Contest the Director, and registered together with
An application is prosecuted ex parte by the the description, claims, and drawings, if
applicant; that is, the proceedings are like a any, in books and records of the Office.
lawsuit in which there is a plaintiff, but no
defendant, the court itself acting as adverse Term
party. IP Code, Section 54
The term of a patent shall be twenty (20)
years from the filing date of the
Amendment of Specification
application.
IP Code, Section 49
An applicant may amend the patent
application during examination: Provided, Note: for computing the term, use the local
That such amendment shall not include filing date even if the applicant made use of
new matter outside the scope of the a priority date in a foreign application
disclosure contained in the application as
filed. Patent After Grant
IP Code, Section 55.
Grant of Patent Annual Fees. —
55.1. To maintain the patent application or
- Ultimate objective of patent application
patent, an annual fee shall be paid upon
Intellectual Property Law ►►| 57

the expiration of four (4) years from the from manufacturing, dealing in,
date the application was published using, selling, or offering for sale, or
pursuant to Section 44 hereof, and on importing any product obtained
each subsequent anniversary of such directly or indirectly from such
date. Payment may be made within three process.
(3) months before the due date. The 2. Assign, or transfer the patent, and
obligation to pay the annual fees shall conclude licensing contracts
terminate should the application be
withdrawn, refused, or cancelled. Cancellation of Patent
IP Code, Section61.
55.2. If the annual fee is not paid, the Cancellation of Patents –
patent application shall be deemed 61.1. Any interested person may, upon
withdrawn or the patent considered as payment of the required fee, petition to
lapsed from the day following the cancel the patent or any claim thereof, or
expiration of the period within which the parts of the claim, on any of the following
annual fees were due. A notice that the grounds:
application is deemed withdrawn or the (a) That what is claimed as the invention
lapse of a patent for non-payment of any is not new or patentable;
annual fee shall be published in the IPO (b) That the patent does not disclose the
Gazette and the lapse shall be recorded in invention in a manner sufficiently clear
the Register of the and complete for it to be carried out by
Office. any person skilled in the art; or
(c) That the patent is contrary to public
55.3. A grace period of six (6) months shall order or morality.
be granted for the payment of the annual
fee, upon payment of the prescribed 61.2. Where the grounds for cancellation
surcharge for delayed payment. relate to some of the claims or parts of
the claim, cancellation may be effected to
The first annual fee is due after the such extent only.
expiration of four years from date of the
publication of the application and on each
subsequent anniversary thereafter. INFRINGEMENT
Payments may be made 3 months before
due date. What constitutes infringement? (Section
76)
Obligation to pay is terminated if application IP Code, Section 76
is withdrawn, refused or cancelled. Civil Action for Infringement. ‑
76.1. The making, using, offering for sale,
RIGHTS CONFERRED BY A PATENT selling, or importing a patented product or
Q: What are the exclusive rights a product obtained directly or indirectly
conferred by a patent? (Section 71) from a patented process, or the use of a
A: The following are the exclusive rights patented process without the
conferred by a patent: authorization of the patentee constitutes
1. Product v. Process patent infringement: Provided, That, this
a. product – to restrain, prohibit and shall not apply to instances covered by
prevent any unauthorized person or Sections 72.1 and 72.4 (Limitations of
entity from making, using, offering for Patent Rights); Section 74 (Use of
sale, selling or importing that product Invention by Government); Section 93.6
b. process – to restrain, prevent or (Compulsory Licensing); and Section 93-
prohibit any unauthorized person or A (Procedures on Issuance of a Special
entity from using the process and
Intellectual Property Law ►►| 58

Compulsory License under the TRIPS Q: Can a contributory infringer be held


Agreement) of this Code. liable?
A: YES. (Section 76.6)
76.2. Any patentee, or anyone possessing IP Code, Section 76.6.
any right, title or interest in and to the Anyone who actively induces the
patented invention, whose rights have infringement of a patent or
been infringed, may bring a civil action provides the infringer with a
before a court of competent jurisdiction, component of a patented product
to recover from the infringer such or of a product produced because
damages sustained thereby, plus of a patented process knowing it
attorney’s fees and other expenses of to be especially adopted for
litigation, and to secure an injunction for infringing the patented invention
the protection of his rights. and not suitable for substantial
non-infringing use shall be liable
Q: May a foreign national bring an action as a contributory infringer and
for infringement of patent, whether or shall be jointly and severally
not it is licensed to do business in the liable with the infringer.
Philippines?
A: Yes. See Sections 3 and 77 Q: Invention kits?
A: May constitute infringement
Q: What if damages are inadequate or
cannot be readily ascertained? (Sections PRESUMPTION IN PROCESS PATENT;
76.3 and 74) SHIFTING THE BURDEN OF PROOF
IP Code Section 76.3. IP Code, Section 78.
If the damages are inadequate or cannot Process Patents; Burden of Proof. ‑
be readily ascertained with reasonable If the subject matter of a patent is a
certainty, the court may award by way of process for obtaining a product, any
damages a sum equivalent to reasonable identical product shall be presumed to
royalty. have been obtained through the use of
the patented process if the product is
new or there is substantial likelihood that
IP Code, Section 76.4. the identical product was made by the
The court may, according to the process and the owner of the patent has
circumstances of the case, award been unable despite reasonable efforts,
damages in a sum above the amount to determine the process actually used. In
found as actual damages sustained: ordering the defendant to prove that the
Provided, That the award does not process to obtain the identical product is
exceed three (3) times the amount of different from the patented process, the
such actual damages. court shall adopt measures to protect, as
far as practicable, his manufacturing and
Q: How do courts dispose infringing business secrets.
goods? (Section 76.5)
IP Code, Section 76.5. PRESCRIPTION IN ACTION FOR
The court may, in its discretion, order INFRINGEMENT
that the infringing goods, materials and IP Code, Section 79.
implements predominantly used in the Limitation of Action for Damages. ‑
infringement be disposed of outside the No damages can be recovered for acts of
channels of commerce or destroyed, infringement committed more than four
without compensation. (4) years before the institution of the
action for infringement.
Intellectual Property Law ►►| 59

IP Code, Section 80. LITERAL INFRINGEMENT


Damages; Requirement of Notice. ‑
Damages cannot be recovered for acts of Godines v. Court of Appeals
infringement committed before the Infringement?
infringer had known, or had reasonable
grounds to know of the patent. It is Tests have been established to determine
presumed that the infringer had known of infringement. These are (a) literal
the patent if on the patented product, or infringement; and (b) the doctrine of
on the container or package in which the equivalents. In using literal infringement as a
article is supplied to the public, or on the test, ". . . resort must be had, in the first
advertising material relating to the instance, to the words of the claim. If accused
patented product or process, are placed matter clearly falls within the claim,
the words “Philippine Patent” with the infringement is made out and that is the end of
number of the patent. it." To determine whether the particular item
falls within the literal meaning of the patent
IP Code, Section 81. claims, the court must juxtapose the claims of
Defenses in Action for Infringement. ‑ the patent and the accused product within the
In an action for infringement, the defendant, overall context of the claims and
in addition to other defenses available to specifications, to determine whether there is
him, may show the invalidity of the patent, exact identity of all material elements.
or any claim thereof, on any of the grounds
on which a petition of cancellation can be (Court observed that the patent and the
brought under Section features of the patented utility model were
61 hereof. copied by the petitioner.)

Q: Can you file a criminal action for The powers tillers are identical in
infringement? configuration, design and appearance.
A: YES. (Section 84) DOCTRINE OF EQUIVALENTS
An infringement also occurs when a device
appropriates a prior invention by
IP Code, Section 84. incorporating its innovative concept and,
Criminal Action for Repetition of albeit with some modification and change,
Infringement. ‑ performs substantially the same function in
If infringement is repeated by the substantially the same way to achieve
infringer or by anyone in connivance with substantially the same result.
him after finality of the judgment of the
court against the infringer, the offenders Carlos Gsell v. Valerioso Veloso Yap-
shall, without prejudice to the institution Jue
of a civil action for damages, be Infringement?
criminally liable therefor and, upon
conviction, shall suffer imprisonment for “The use of a process in all respects
the period of not less than six (6) months identical with a process protected by a valid
but not more than three (3) years and/or a patent, save only that a well-known
fine of not less than One hundred mechanical equivalent is substituted in lieu
thousand pesos (P100,000) but not more of some particular part of the patented
than Three hundred thousand pesos process is an infringement upon the rights
(P300,000), at the discretion of the court. of the owner of the patent, which will be
The criminal action herein provided shall enjoined in appropriate proceeding…”
prescribe in three (3) years from date of
the commission of the crime.
Intellectual Property Law ►►| 60

Q: Is the infringing feature (or claim) Market Presence


insubstantial or not? (as used in the case IP Code, Section 72.1.
“unessential change [in the feature]?)” Using a patented product which has been
A: NO. Doctrine of mechanical equivalents put on the market in the Philippines by
was applied such that the patentee is the owner of the product, or with his
protected from colorable invasions of his express consent, insofar as such use is
patent under the guise of a substitution of performed after that product has been so
some part of his invention by some well- put on the said market: Provided, That,
known mechanical equivalent. with regard to drugs and medicines, the
limitation on patent rights shall apply
Smithkline Beckman Corporation v. after a drug or medicine has been
Court of Appeals, August 14, 2003 introduced in the Philippines or anywhere
No infringement? else in the world by the patent owner, or
by any party authorized to use the
The doctrine of equivalents thus requires invention: Provided, further, That the
satisfaction of the function-means-and- right to import the drugs and medicines
result test, the patentee having the burden contemplated in this section shall be
to show that all 3 components of such available to any government agency or
equivalency test are met any private third party;

(“Performs substantially the same function Act is done privately and on a


in substantially the same way, yield noncommercial scale
substantially the same result ”) IP Code, Section 72.2.
Where the act is done privately and on a
non-commercial scale or for a non-
commercial purpose: Provided, That it
DOCTRINE OF EQUIVALENTS does not significantly prejudice the
LIMITATION: economic interests of the owner of the
patent;
In the US, the doctrine of equivalents is
limited by prosecution history estoppel.
Acts done for experimental
Under the prosecution history estoppel, if purposes IP Code, Section 72.3
72.3. Where the act consists of making or
the patentee abandoned through an
using exclusively for experimental use of
amendment to the patent application certain
the invention for scientific purposes or
literal claim coverage (e.g. by narrowing the
educational purposes and such other
literal scope of the patent claim), the
activities directly related to such scientific
patentee is estopped from later arguing that
or educational experimental use;
the surrendered coverage is insubstantially
different from the literally claimed limitation.
Testing and using related to issuance of
approvals by government regulatory
LIMITATIONS ON PATENT RIGHTS agencies
IP Code, Section 72.4.
In the case of drugs and medicines, where
IP Code, Section 72.
the act includes testing, using, making or
Limitations of Patent Rights. ‑
selling the invention including any data
The owner of a patent has no right to related thereto, solely for purposes
prevent third parties from performing,
reasonably related to the development and
without his authorization, the acts
submission of information and issuance of
referred to in Section 71 hereof in the
approvals by government regulatory
following circumstances:
agencies required under any
Intellectual Property Law ►►| 61

law of the Philippines or of another country determined by the appropriate agency


that regulates the manufacture, of the government, so requires; or
construction, use or sale of any product: (b) A judicial or administrative body has
Provided, That, in order to protect the data determined that the manner of
submitted by the original patent holder exploitation, by the owner of the
from unfair commercial use provided in patent or his licensee is anti-
Article 39.3 of the Agreement on Trade- competitive; or
Related Aspects of Intellectual Property (c) In the case of drugs and medicines,
Rights (TRIPS Agreement), the Intellectual there is a national emergency or other
Property Office, in consultation with the circumstance of extreme urgency
appropriate government agencies, shall requiring the use of the invention; or
issue the appropriate rules and regulations (d) In the case of drugs and medicines,
necessary therein not later than one there is public non-commercial use
hundred twenty (120) days after the of the patent by the patentee, without
enactment of this law; satisfactory reason; or
(e) In the case of drugs and medicines, the
Medical prescription for individual cases demand for the patented article in the
IP Code Section 72.5. Philippines is not being met to an
Where the act consists of the preparation adequate extent and on reasonable
for individual cases, in a pharmacy or by terms, as determined by the Secretary
a medical professional, of a medicine in of the Department of Health.
accordance with a medical prescription
or acts concerning the medicine so IP Code, Section 74.2
prepared; and Unless otherwise provided herein, the
use by the Government, or third person
In Transit authorized by the Government shall be
IP Code, Section 72.6 subject, where applicable, to the
Where the invention is used in any ship, following provisions:
vessel, aircraft, or land vehicle of any
other country entering the territory of the (a) In situations of national emergency or
Philippines temporarily or accidentally: other circumstances of extreme urgency
Provided, That such invention is used as provided under Section 74.1(c), the
exclusively for the needs of the ship, right holder shall be notified as soon as
vessel, aircraft, or land vehicle and not reasonably practicable;
used for the manufacturing of anything to
be sold within the Philippines. (b) In the case of public non-commercial
use of the patent by the patentee, without
ADDITIONAL LIMITATIONS ON satisfactory reason, as provided under
Section 74.1 (d), the right holder shall be
PATENT RIGHTS informed promptly: Provided, That, the
Government or third person authorized by
Use of Invention by Government the Government, without making a patent
IP Code, Section 74. search, knows or has demonstrable ground
Use of Invention by Government. ‑ 74.1. to know that a valid patent is or will be used
A Government agency or third person by or for the Government;
authorized by the Government may
exploit the invention even without (c) If the demand for the patented article
agreement of the patent owner where: in the Philippines is not being met to an
(a) The public interest, in particular, adequate extent and on reasonable terms
national security, nutrition, health or as provided under Section 74.1 (e), the
the development of other sectors, as right holder shall be informed promptly;
Intellectual Property Law ►►| 62

IP Code, Section 74.3. Bureau shall exercise quasi-judicial


All cases arising from the implementation jurisdiction in the settlement of disputes
of this provision shall be cognizable by between parties to a technology transfer
courts with appropriate jurisdiction arrangement arising from technology
provided by law. transfer payments, including the fixing of
appropriate amount or rate of royalty.
No court, except the Supreme Court of the
Philippines, shall issue any temporary IP Code, Section 87.
restraining order or preliminary injunction Prohibited Clauses. ‑
or such other provisional remedies that will Except in cases under Section 91, the
prevent its immediate execution. following provisions shall be deemed
prima facie to have an adverse effect on
Review on Patents: competition and trade:
1. The legal consequence(s) when the
provisions of a Technology Transfer 87.1. Those which impose upon the
Arrangement is inconsistent with the licensee the obligation to acquire from a
provisions of Sec. 87 and 88 of the IP specific source capital goods,
Code. intermediate products, raw materials, and
2. The rules applicable when a third party other technologies, or of permanently
is entitled to use patented subject matter employing personnel indicated by the
licensor;
without the consent of the owner of the
patent:
87.2. Those pursuant to which the
a. Use is covered by the Limitations on licensor reserves the right to fix the sale
Patent Rights or resale prices of the products
b. Use of invention by the Government manufactured on the basis of the license;
c. Third party is a prior user in good faith;
and 87.3. Those that contain restrictions
d. The reason for the use is among the regarding the volume and structure of
grounds for Compulsory Licensing production;

VOLUNTARY LICENSING 87.4. Those that prohibit the use of


competitive technologies in a non-
IP Code, Section 85. exclusive technology transfer agreement;
Voluntary License Contract. –
To encourage the transfer and 87.5. Those that establish a full or partial
dissemination of technology, prevent or purchase option in favor of the licensor;
control practices and conditions that may
in particular cases constitute an abuse of 87.6. Those that obligate the licensee to
intellectual property rights having an transfer for free to the licensor the
adverse effect on competition and trade, inventions or improvements that may be
all technology transfer arrangements obtained through the use of the licensed
shall comply with the provisions of this technology;
Chapter.
87.7. Those that require payment of
IP Code, Section 86. royalties to the owners of patents for
patents which are not used;
Jurisdiction to Settle Disputes on
Royalties. –
87.8. Those that prohibit the licensee to
The Director of the Documentation,
export the licensed product unless
Information and Technology Transfer
Intellectual Property Law ►►| 63

justified for the protection of the legitimate BAR Q: What contractual stipulations are
interest of the licensor such as exports to required in all technology transfer
countries where exclusive licenses to agreements?
manufacture and/or distribute the licensed A: The following are the stipulations
product(s) have already been granted; required:

87.9. Those which restrict the use of the IP Code, Section 88.
technology supplied after the expiration Mandatory Provisions. ‑
of the technology transfer arrangement, The following provisions shall be
except in cases of early termination of the included in voluntary license contracts:
technology transfer arrangement due to
reason(s) attributable to the licensee; 88.1. That the laws of the Philippines shall
govern the interpretation of the same and in
87.10. Those which require payments for the event of litigation, the venue shall be
patents and other industrial property the proper court in the place where the
rights after their expiration, termination licensee has its principal office;
arrangement;
88.2. Continued access to improvements
87.11. Those which require that the in techniques and processes related to
technology recipient shall not contest the the technology shall be made available
validity of any of the patents of the during the period of the technology
technology supplier; transfer arrangement;

87.12. Those which restrict the research 88.3. In the event the technology transfer
and development activities of the licensee arrangement shall provide for arbitration,
designed to absorb and adapt the the Procedure of Arbitration of the
transferred technology to local conditions Arbitration Law of the Philippines or the
or to initiate research and development Arbitration Rules of the United Nations
programs in connection with new products, Commission on International Trade Law
processes or equipment; (UNCITRAL) or the Rules of Conciliation
and Arbitration of the International
87.13. Those which prevent the licensee Chamber of Commerce (ICC) shall apply
from adapting the imported technology to and the venue of arbitration shall be the
local conditions, or introducing Philippines or any neutral country; and
innovation to it, as long as it does not
impair the quality standards prescribed 88.4. The Philippine taxes on all
by the licensor; payments relating to the technology
transfer arrangement shall be borne by
87.14. Those which exempt the licensor the licensor.
for liability for non-fulfilment of his
responsibilities under the technology IP Code, Section 91.
transfer arrangement and/or liability Exceptional Cases. ‑
arising from third party suits brought In exceptional or meritorious cases where
about by the use of the licensed product substantial benefits will accrue to the
or the licensed technology; and economy, such as high technology
content, increase in foreign exchange
87.15. Other clauses with equivalent earnings, employment generation,
effects. regional dispersal of industries and/or
substitution with or use of local raw
materials, or in the case of Board of
Intellectual Property Law ►►| 64

Investments, registered companies with 93.2. Where the public interest, in


pioneer status, exemption from any of the particular, national security, nutrition,
above requirements may be allowed by health or the development of other vital
the Documentation, Information and sectors of the national economy as
Technology Transfer Bureau after determined by the appropriate agency of
evaluation thereof on a case by case the Government, so requires; or
basis.
93.3. Where a judicial or administrative
IP Code, Section 92. body has determined that the manner of
Non-Registration with the exploitation by the owner of the patent or
Documentation, Information and his licensee is anti-competitive; or
Technology Transfer Bureau. ‑
Technology transfer arrangements that 93.4. In case of public non-commercial
conform with the provisions of Sections use of the patent by the patentee, without
86 and 87 need not be registered with the satisfactory reason;
Documentation, Information and
Technology Transfer Bureau. Non- 93.5. If the patented invention is not being
conformance with any of the provisions worked in the Philippines on a
of Sections 87 and 88, however, shall commercial scale, although capable of
automatically render the technology being worked, without satisfactory
transfer arrangement unenforceable, reason: Provided, That the importation of
unless said technology transfer the patented article shall constitute
arrangement is approved and registered working or using the patent. (Secs. 34,
with the Documentation, Information and 34-A, 34-B, R.A. No. 165a) and
Technology Transfer Bureau under the
provisions of Section 91 on exceptional 93.6. Where the demand for patented
cases. drugs and medicines is not being met to
an adequate extent and on reasonable
Q: Is an exclusive licensing arrangement terms, as determined by the Secretary of
a Prohibited Act under the Philippine the Department of Health.
Fair Competition Act?
A: No. This is a recognized exception to the SPECIAL COMPULSORY LICENSE
general rule. (The Philippine Competition UNDER THE TRIPS AGREEMENT
Act, Sec. 2 and 3 Chapter III) (SECTION 93A)
IP Code Section 93-A.
Procedures on Issuance of a Special
COMPULSORY LICENSING Compulsory License under the TRIPS
Agreement. ‑
IP Code, Section 93.
Section 93-A.1. The Director General of
Grounds for Compulsory Licensing. ‑
the Intellectual Property Office, upon the
The Director General of the Intellectual
written recommendation of the Secretary
Property Office may grant a license to
of the Department of Health, shall, upon
exploit a patented invention, even without filing of a petition, grant a special
the agreement of the patent owner, in compulsory license for the importation of
favor of any person who has shown his patented drugs and medicines. The
capability to exploit the invention, under special compulsory license for the
any of the following circumstances: importation contemplated under this
provision shall be an additional special
93.1. National emergency or other alternative procedure to ensure access to
circumstances of extreme urgency; quality affordable medicines and shall be
Intellectual Property Law ►►| 65

primarily for domestic consumption: Smith Kline & French Lab Ltd. V. Court
Provided, That adequate remuneration shall of Appeals, July 24, 1997
be paid to the patent owner either by the Q: Whether Smith Kline was deprived of its
exporting or importing country. The property right with the grant of the
compulsory license shall also contain a Compulsory License:
provision directing the grantee the license A: NO. Smith Kline would be entitled to
to exercise reasonable measures to prevent receive royalties.
the re-exportation of the products imported
under this provision. Smith Kline & French Lab Ltd. V. Court
of Appeals, Oct. 23, 2001
The grant of a special compulsory license
Q: Whether the royalty rate under the
under this provision shall be an
Compulsory License is unreasonable.
exception to Sections 100.4 and 100.6 of
A: NO. The Director General is given
Republic Act No. 8293 and shall be
discretion to award royalty not more than
immediately executory.
5% of net wholesale price. The 2.5% royalty
is reasonable especially because there is no
No court, except the Supreme Court of
showing that petitioner granted technical
the Philippines, shall issue any temporary
assistance to the respondent.
restraining order or preliminary
injunction or such other provisional
remedies that will prevent the grant of the
special compulsory license. UTILITY MODELS
Q: Whether incidental matters such as
issuance of an injunction should be - Petty inventions
brought before the IPO Director General - As compared to patents, utility models
or the Court of Appeals: lack the element of inventive step

Philippine Pharmawealth v. Pfizer IP Code Section 108


Final orders – to the Director General Applicability of Provisions Relating to
Patents. —
Interlocutory Orders – to the Court of 108.1. Subject to Section 109, the
Appeals provisions governing patents shall apply,
mutatis mutandis, to the registration of
Roma Drug v. RTC
utility models.
Q: Parallel Importation involves genuine
goods. 108.2. Where the right to a patent conflicts
with the right to a utility model registration
IP Code allows parallel importation of in the case referred to in Section 29, the
patented medicines. said provision shall apply as if the word
"patent" were replaced by the words
Perke Davis & Co. v. Doctor’s "patent or utility model registration".
Pharmaceuticals Inc. et al
Q: What is the ground for filing for IP Code Section 109
Compulsory License in this case? Special Provisions Relating to Utility
A: Public interest Models. —
109.1.
a. An invention qualifies for registration
as a utility model if it is new and
industrially applicable.
Intellectual Property Law ►►| 66

b. Section 21, "Patentable Inventions", composition or form gives a special


shall apply except the reference to appearance to and can serve as pattern
inventive step as a condition of for an industrial product or handicraft.
protection. (Sec. 55, R.A. No. 165a)

109.2. Sections 43 to 49 shall not apply in


the case of applications for registration of IP Code, Section 118.
a utility model. The Term of Industrial Design
Registration. —
109.3. A utility model registration shall 118.1. The registration of an industrial
expire, without any possibility of renewal, design shall be for a period of five
at the end of the seventh year after the (5) years from the filing date of the
date of the filing of the application. application.

109.4. In proceedings under Sections 61 118.2. The registration of an industrial


to 64, the utility model registration shall design may be renewed for not more than
be canceled on the following grounds: two (2) consecutive periods of five (5)
a. That the claimed invention does not years each, by paying the renewal fee.
qualify for registration as a utility The renewal fee shall be paid within
model and does not meet the twelve (12) months preceding the
requirements of registrability, in expiration of the period of registration.
particular having regard to
Subsection 109.1 and Sections 22, However, a grace period of six (6) months
23, 24 and 27; shall be granted for payment of the fees
b. That the description and the claims after such expiration, upon payment of a
do not comply with the prescribed surcharge
requirements;
c. That any drawing which is necessary 118.3. The Regulations shall fix the
for the understanding of the amount of renewal fee, the surcharge and
invention has not been furnished; other requirements regarding the
d. That the owner of the utility model recording of renewals of registration.
registration is not the inventor or his
successor in title. (Secs. 55, 56, and IP Code, Section 120.
57, R.A. No. 165a)
Cancellation of Design Registration. –
120.1. At any time during the term of the
Ching v. Salinas industrial design registration, any person
While works of applied art, original upon payment of the required fee, may
intellectual, literary and artistic works are petition the Director of Legal Affairs to
copyrightable, useful articles and works of cancel the industrial design on any of the
industrial design are not. following grounds:
a. If the subject matter of the
industrial design is not registrable
INDUSTRIAL DESIGNS within the terms of Sections 112
and 113;
IP Code, Section 112. b. If the subject matter is not new; or
Definition of Industrial Design. – c. If the subject matter of the
An industrial design is any composition of industrial design extends beyond
lines or colors or any three-dimensional the content of the application as
form, whether or not associated with lines originally filed.
or colors: Provided, That such
Intellectual Property Law ►►| 67

120.2. Where the grounds for cancellation


relate to a part of the industrial design,
cancellation may be effected to such
extent only. The restriction may be
effected in the form of an alteration of the
effected features of the design. (n)

Note: for utility models and industrial


designs, substantive examination is not
required

UTILITY MODEL AND INDUSTRIAL


DESIGN REGISTRATION PROCEDURE
(see chart at the last page)

Apple Inc. v. Samsung Electronics Co.


Ltd
1. 647 Patent – Quick links feature
2. 721 Patent – Slide to unlock feature
What were the two prior art references
cited in the Decision? What was the
problem 721 intended to resolve?
- Neonode (swipe right to unlock) and
Plaisant (toggle design)
- The problem addressed by the 721
patent is pocket dialing
3. 172 Patent - Autocorrect
What were the two prior art
references cited in the Decision?
How did the Court resolve that the
invention was not obvious (from the
prior inventions)?
- Robinson (keyboard system with
suggestions on misspelt words) and
Xrgomics (word completion patent –
makes suggestion of words once
you type letters)
- The court resorted to applying the
Graham factors; and found that not
all the elements of the claim are
present in the combination of the two
prior arts: Robinson and Xrgomics
Intellectual Property Law ►►| 68

Figure 1: How Madrid Protocol Works

Figure 2: Patent Application Flowchart


Intellectual Property Law ►►| 69

Figure 3: Utility Model and Industrial Design Registration Procedure

Vous aimerez peut-être aussi